Sei sulla pagina 1di 67

American Academy of Orthopaedic Surgeons

2004 pediatric orthopedic Surgery

1. An 8-year-old boy sustains nondisplaced midshaft fractures of the tibia and fibula
after being struck by a car while he was riding his bicycle. No other injuries are
noted, but the patient reports pain with passive motion of his toes. His
neurovascular examination is otherwise normal. What is the best course of action?

1- Hospital admission with a referral to social services for evaluation for child neglect
2- Long leg casting and reevaluation in 24 hours
3- Short leg casting and reevaluation in 24 hours
4- Electrical studies of nerve function in 3 to 6 weeks if there is no improvement
5- Compartment pressure measurements and inpatient treatment as indicated

PREFERRED RESPONSE: 5

DISCUSSION: Pain with passive motion of the toes is a recognized early sign of increased
compartment pressures. At a minimum, a baseline evaluation of the leg compartment pressures
should be obtained. While it is normal for the patient to have pain related to the associated
muscle contusions, any significant concerns should be addressed immediately in light of the
severe consequences likely when a compartment syndrome occurs.
 Mubarak SJ, Owen CA, Hargens AR, et al: Acute compartment syndromes: Diagnosis and treatment with the aid of the wick
catheter. J Bone Joint Surg Am 1978;60:1091-1095.
 Matsen FA III, Veith RG: Compartmental syndromes in children. J Pediatr Orthop 1981;1:33-41.

2. A 6-year-old girl has the bilateral foot deformity shown in Figure 1. There is no
family history of disease. Examination reveals fixed hindfoot equinus, and muscle
function testing shows strong posterior tibial function, fair plus anterior tibial
function, poor peroneal function, and strong gastrocnemius function. A Coleman
block test shows a correctable hindfoot. Nerve conduction velocity studies show
diminished function in the peroneal and ulnar nerves on both sides. Pathologic
changes found in a sural nerve biopsy include “onion bulb” formation, and DNA
testing confirms the presence of a mutation in the MPZ gene, consistent with
hereditary motor sensory neuropathy type III (HMSN-III). What is the best course
of action?

1- Triple arthrodesis
2- Midfoot closing wedge
resection osteotomy
3- Calcaneal and
metatarsal osteotomies
4- Plantar release,
posterior tibial tendon
transfer to the dorsum,
and Achilles tendon
lengthening
5- Serial casting
American Academy of Orthopaedic Surgeons
2004 pediatric orthopedic Surgery

PREFERRED RESPONSE: 4

DISCUSSION: The patient has HMSN-III or Dejerine-Sottas syndrome. This form of HMSN
progresses very rapidly and frequently results in severe foot deformity in early childhood. The
changes are progressive and are the result of muscle imbalance during growth. Balancing of the
foot musculature is essential, particularly during the phases of rapid growth of the foot.
However, this cannot be accomplished using the anterior tibial muscle because it is already weak
and the transfer will further weaken it. Bony procedures also may be required, and tendon
transfers cannot be depended on to correct bony deformity. However, these procedures can be
deferred until the foot is closer to adult size. Surgeries that lead to joint arthrodesis, such as
triple arthrodesis and some midfoot osteotomies, are contraindicated because the feet may lose
protective sensation as the disease progresses. Fusions in insensate feet are less successful than
realignment procedures that maintain mobility.
 Wetmore RS, Drennan JC: Long-term results of triple arthrodesis in Charcot-Marie-Tooth disease. J Bone Joint Surg Am
1989;71:417-422.
 Roper BA, Tibrewal SB: Soft tissue surgery in Charcot-Marie-Tooth. J Bone Joint Surg Br 1989;71:17-20.
 Miller GM, Hsu JD, Hoffer MM, Rentfro R: Posterior tibial tendon transfer: A review of the literature and analysis of 74
procedures. J Pediatr Orthop 1982;2:363-370.

3. An obese 4-year-old boy has infantile Blount’s disease. Radiographs reveal a


metaphyseal-diaphyseal angle of 18 degrees and a depression of the medial proximal
tibial physis. Management should consist of

1- observation.
2- varus prevention orthoses.
3- physeal bar resection.
4- proximal tibial osteotomy that produces a neutral mechanical axis.
5- proximal tibial osteotomy that produces 10 degrees of valgus.

PREFERRED RESPONSE: 5

DISCUSSION: The deformity is too severe for observation, and at age 4 years, the child is too
old for orthotic treatment. To prevent recurrence, surgery should be performed before
irreversible changes occur in the medial physis. A proximal tibial osteotomy should overcorrect
the mechanical axis to 10 degrees of valgus. Bar resection has not been shown to be as effective
in this severe deformity, especially without a concomitant osteotomy.
 Raney EM, Topoleski TA, Yaghoubian R, Guidera KJ, Marshall JG: Orthotic treatment of infantile tibia vara. J Pediatr
Orthop 1998;18:670-674.
 Loder RT, Johnston CE: Infantile tibia vara. J Pediatr Orthop 1987;7:639-646.

4. A 10-year-old boy has activity-related knee pain that is poorly localized. He denies
locking, swelling, or giving way. Examination shows mild tenderness at the medial
femoral condyle and painless full range of motion without ligamentous instability.
Radiographs are shown in Figures 2a through 2c. What is the best course of action?
American Academy of Orthopaedic Surgeons
2004 pediatric orthopedic Surgery

1- Knee arthroscopy with drilling of the lesion


2- Limited activity for 6 to 12 weeks
3- Removal of the loose body
4- Biopsy of the lesion
5- Open reduction and internal fixation

PREFERRED RESPONSE: 2

DISCUSSION: The radiographs show an osteochondritis dissecans (OCD) lesion in the medial
femoral condyle of a skeletally immature patient. The lesion is not displaced from its bed.
Nonsurgical management of a stable OCD lesion in a patient with open physes consists of a
period of activity limitation and occasional immobilization. Unstable lesions, loose bodies, and
patients with closed physes require more aggressive treatment. Most of the surgical procedures
can be done arthroscopically. Because the radiographic appearance is typical, biopsy is
unnecessary. The radiographs do not show an osteocartilaginous loose body, and the patient
reports no catching or locking; therefore, removal of the loose body is not indicated.
 Linden B: Osteochondritis dissecans of the femoral condyles: A long term follow-up study. J Bone Joint Surg Am
1977;59:769-776.
 Cahill BR: Osteochondritis dissecans of the knee: Treatment of juvenile and adult forms. J Am Acad Orthop Surg
1995;3:237-247.
 Cahill BR, Navarro R: The results of conservative management of juvenile osteochondritis dissecans using joint
scintigraphy: A prospective study. Am J Sports Med 1989;17:601-606.

5. Figure 3a shows the preoperative


radiograph of a 5-year-old girl who
achieved complete correction with valgus
osteotomies. Figure 3b shows a
radiograph obtained 2 years later. What
is the cause of the recurrent deformity on
the right side?

1- Inadequate restoration of the weight-


bearing axis
American Academy of Orthopaedic Surgeons
2004 pediatric orthopedic Surgery

2- Partial growth arrest of the medial tibial physis


3- Age older than 3 years
4- Obesity (greater than the 95th percentile)
5- A metaphyseal-diaphyseal angle of greater than 20 degrees

PREFERRED RESPONSE: 2

DISCUSSION: Although inadequate correction, obesity, patient age of older than 5 years and an
increased metaphyseal-diaphyseal angle are all associated with a poorer outcome, the
radiographs show a growth arrest of the medial tibial physis. If not recognized and treated with
early surgery, progressive genu varum will occur with continued growth of the lateral physis. In
addition to repeat osteotomy, options for treating the arrest include physeal bar resection or, as
necessary, completion of the growth arrest by epiphyseodesis of the lateral physes, followed by a
limb equalization procedure at a later date.
 Brooks WC, Gross RH: Genu varum in children: Diagnosis and treatment. J Am Acad Orthop Surg 1995;3:326-335.
 Herring JA: Tachdjian's Pediatric Orthopedics, ed 4. Philadelphia, PA, WB Saunders, 2002, pp 840-950.
 Schoenecker PL, Rich MM: The lower extremity, in Morrissy RT, Weinstein SL (eds): Lovell and Winter's Pediatric
Orthopaedics, ed 5. Philadelphia, PA, Lippincott Williams and Wilkins, 2001, pp 1068-1073.

6. An 8-year-old boy reports ankle pain after striking the ground with the medial
aspect of his foot while attempting to kick a soccer ball. Radiographs reveal slight
distal tibial physeal widening but no other abnormalities. In treating this injury,
which of the following associated conditions is most likely present but may be
missed without careful evaluation?

1- Malrotation of the foot


2- Neurologic injury
3- Vascular injury
4- Knee meniscal injury
5- Hip fracture

PREFERRED RESPONSE: 1

DISCUSSION: Malrotation of the foot is frequently overlooked in this clinical setting. This can
be judged by evaluating and comparing the transmalleolar axes of the affected and unaffected
legs. The rotation occurs through the physis and frequently is not recognized until the patient
has been walking for a few months. The other conditions are not expected to occur in the
clinical setting described.
 Phan VC, Wroten E, Yngve DA: Foot progression angle after distal tibial physeal fractures. J Pediatr Orthop 2002;22:31-
35.
 Brook GJ, Greer RB: Traumatic rotational displacements of the distal tibial growth plate. J Bone Joint Surg Am
1970;52:1666-1668.
American Academy of Orthopaedic Surgeons
2004 pediatric orthopedic Surgery

7. An 11-year-old girl has wrist pain. Figure 4a shows the radiograph, and Figures 4b
and 4c show the low- and medium-power photomicrographs of a lesion in the distal
radius. What is the most likely diagnosis?

1- Simple bone cyst


2- Nonossifying fibroma
3- Osteomyelitis
4- Aneurysmal bone cyst
5- Giant cell tumor

PREFERRED RESPONSE: 4

DISCUSSION: The radiograph shows an osteolytic eccentric lesion in the metaphyseal-


diaphyseal region of the bone, and the photomicrographs show an aneurysmal bone cyst. The
low-power photomicrograph shows large empty spaces with fibrous stroma and multinucleated
giant cells. The red area in the center is hemorrhage in the stroma. The large empty spaces are
cysts, which would be filled with blood in vivo. The medium-power photomicrograph shows a
large cyst-like space and hemorrhage in the surrounding stoma. Giant cell tumors have "sheets"
of giant cells. A nonossifying fibroma would have spindle cells, and a unicameral bone cyst may
have a few giant cells, but blood is rare.
 Springfield DS, Gebhardt MC: Bone and soft tissue tumors, in Morrissy RT, Weinstein SL (eds): Lovell and Winter's
Pediatric Orthopaedics, ed 5. Philadelphia, PA, Lippincott Williams and Wilkins, 2001, pp 540-542.
 Herring JA: Tachdjian's Pediatric Orthopedics, ed 4. Philadelphia, PA, WB Saunders, 2002, pp 1906-1909.

8. In a patient with vertebral tuberculosis, which of the following characteristics is


most predictive of progression of the kyphosis?

1- Involvement of the vertebral body and posterior elements


2- Involvement of the thoracic vertebrae
3- Involvement of the lumbar vertebrae
4- Age of the patient
5- Pretreatment degree of kyphosis
American Academy of Orthopaedic Surgeons
2004 pediatric orthopedic Surgery

PREFERRED RESPONSE: 1

DISCUSSION: In patients with vertebral tuberculosis, involvement of the anterior and posterior
elements creates an instability and severe kyphotic collapse can occur. This characteristic has
been shown to have a stronger association than level of involvement, age, or pretreatment degree
of deformity. In the absence of instability, anterior growth can resume after treatment, leading to
a decrease in the deformity.
 Rajasekaran S: The natural history of post-tubercular kyphosis in children: Radiological signs which predict late increase in
deformity. J Bone Joint Surg Br 2001;83:954-962.
 Rajasekaran S, Shanmagasundaram TK, Prabhakar R, Dheenadhayalan J, Shetty AP, Shetty DK: Tuberculous lesions of the
lumbosacral region: A 15-year follow-up of patients treated by ambulant chemotherapy. Spine 1998;23:1163-1167.

9. When planning scoliosis surgery for a patient with a 50-degree thoracolumbar curve
and spinal muscular atrophy, it is most important to include

1- An anterior release and fusion.


2- A diaphragmatic pacer to assist postoperative pulmonary function.
3- A preoperative gait analysis.
4- An evaluation for lower extremity muscle contractures.
5- Assessment of muscle biopsy findings obtained within the last 6 months to clarify
the patient's life expectancy and thus the value of the surgery.

PREFERRED RESPONSE: 4

DISCUSSION: Typically, posterior spinal fusion to the pelvis is recommended for patients with
spinal muscular atrophy and advanced scoliosis. Examination for lower extremity muscle
contractures is important because the contractures may interfere with good sitting balance.
Anterior release and fusion usually are not advised. Diaphragmatic pacing is not indicated
because diaphragm function usually is not affected. Patients with spinal muscular atrophy
usually are not ambulatory or only marginally ambulatory at the time of scoliosis surgery;
therefore, gait analysis usually is not relevant. While a muscle biopsy may have a role in the
diagnosis of this disorder, it plays no subsequent role in determining life expectancy or the value
of spinal surgery.
 Daher YH, Lonstein JE, Winter RB, Bradford DS: Spinal surgery in spinal muscular atrophy. J Pediatr Orthop 1985;5:391-
395.
 Aprin H, Bowen JR, MacEwen GD, et al: Spinal arthrodesis in patients with spinal muscle atrophy. J Bone Joint Surg Am
1982;64:1179-1187.

10. An 8-year-old boy sustains injuries to his head, abdomen, and left lower extremity
after being struck by a truck. In the emergency department, his mental status
deteriorates and he is intubated after assessment reveals a Glasgow Coma Scale
score of 3; the score subsequently improves to 10. A CT scan reveals a right parietal
intracranial hemorrhage, and an abdominal ultrasound reveals free fluid. Prior to
an emergency laparotomy, the swollen left thigh is evaluated. Radiographs reveal a
American Academy of Orthopaedic Surgeons
2004 pediatric orthopedic Surgery

transverse fracture of the mid-diaphysis. Management of the fracture should


consist of
1- immediate application of a hip spica cast.
2- insertion of a distal femoral traction pin and placement into 90-90 traction.
3- closed reduction and stabilization using retrograde flexible intramedullary nails.
4- insertion of an antegrade reamed interlocking intramedullary nail.
5- closed reduction and transcutaneous pin fixation supplemented by a long leg cast.

PREFERRED RESPONSE: 3

DISCUSSION: The prognosis for a young patient with a head injury is more favorable compared
to that for adults. Full neurologic recovery generally occurs. Spasticity may occur within a few
days after injury, which can lead to fracture displacement if immediate spica casting or traction is
used. Early surgical stabilization will reduce problems with shortening and malunion and will
facilitate transportation of the child for diagnostic tests. Surgery may be performed when it is
best for the patient, either on the day of injury or later if time is needed for stabilization. In this
patient, the fracture is ideally suited to stabilization using flexible intramedullary nails. Heinrich
and associates' report of 78 diaphyseal femur fractures stabilized with flexible intramedullary
nails included 14 patients with an associated closed head injury. All fractures healed, and there
were no major complications.
 Tolo VT: Management of the multiply injured child, in Rockwood CA, Wilkins KE, Beaty JH (eds): Fractures in Children,
ed 4. Philadelphia, PA, Lippincott-Raven, 1996, pp 83-95.
 Heinrich SD, Drvaric DM, Darr K, MacEwen GD: The operative stabilization of pediatric diaphyseal femur fractures with
flexible intramedullary nails: A prospective analysis. J Pediatr Orthop 1994;14:501-507.

11. A 3-year-old boy has a rigid 40-degree lumbar scoliosis that is the result of a fully
segmented L5 hemivertebra. All other examination findings are normal.
Management should consist of

1- in situ posterior fusion.


2- hemivertebral resection and fusion.
3- convex hemiepiphyseodesis.
4- observation with follow-up in 6 months.
5- thoracolumbosacral orthosis bracing.

PREFERRED RESPONSE: 24

DISCUSSION: Near complete correction and rebalancing of the spine can be achieved by
hemivertebral resection that may be done as either a simultaneous or a staged procedure in the
young patient. This eliminates the problem of future progression and possible development of
compensatory curves. Nonsurgical management is not indicated in congenital scoliosis. Convex
hemiepiphyseodesis is best suited for patients younger than age 5 years who have a short curve
caused by fully segmented hemivertebrae that correct to less than 40 degrees with the patient
supine. Hemiepiphyseodesis and isolated posterior fusion are not indicated.
 Bradford DS, Boachie-Adjei O: One-stage anterior and posterior hemivertibral resection and arthrodesis for congenital
scoliosis. J Bone Joint Surg Am 1990;72:536-540.
American Academy of Orthopaedic Surgeons
2004 pediatric orthopedic Surgery

 Lazar RD, Hall JE: Simultaneous anterior and posterior hemivertebra excision. Clin Orthop 1999;364:76-84.

12. A newborn with bilateral talipes equinovarus undergoes serial manipulation and
casting. What is the primary goal of manipulation?

1- Rotation of the foot laterally around the fixed talus


2- Simultaneous abduction of the metatarsals and dorsiflexion of the talus
3- Lateral translation of the calcaneus
4- Anterolateral translation of the navicular
5- Dorsiflexion of the calcaneus with forefoot eversion

PREFERRED RESPONSE: 1

DISCUSSION: Manipulative treatment and casting of talipes equinovarus has become popular
because of disappointing surgical results and enthusiasm for the Ponseti method of manipulation.
In this technique, the primary goal is to rotate the foot laterally around a talus that is held fixed
by the manipulating surgeon's hands. While the navicular may be rotated anterolaterally with
this technique, the primary focus is on the calcaneus. The calcaneus is rotated laterally and
superiorly, not translated. Some dorsiflexion of the calcaneus can be obtained by manipulation,
but the primary focus is on the rotational relationship of the talus and calcaneus, not the degree
of calcaneal dorsiflexion.
 Ponseti IV: Common errors in the treatment of congenital clubfoot. Int Orthop 1997;21:137-141.
 Ponseti IV, Smoley EU: Congenital club foot: The results of treatment. J Bone Joint Surg Am 1963;46:261-275.

13. Figure 5 shows the radiograph of a 10-year-old


girl who reports chronic shoulder pain after
her gymnastics classes. Examination reveals
pain on internal and external rotation but no
instability. What is the most likely diagnosis?

1- Acromial fracture
2- Humeral stress fracture
3- Acromioclavicular joint separation
4- Fracture of the surgical neck of the scapula
5- Triceps avulsion fracture

PREFERRED RESPONSE: 2

DISCUSSION: The patient has a very wide humeral growth plate, indicating the presence of a
proximal humeral stress fracture, an uncommon diagnosis in gymnasts. Gymnasts are prone to
stress fractures of the scaphoid, distal radius, elbow, and clavicle. Proximal humeral stress
American Academy of Orthopaedic Surgeons
2004 pediatric orthopedic Surgery

fractures are more commonly seen in those participating in racket or throwing sports. Stress
fractures can lead to growth arrest or inhibition, particularly in the distal radius. The radiograph
shows normal findings for the acromion, acromioclavicular joint, scapula, and triceps origin.
 Fallon KE, Fricker PA: Stress fracture of the clavicle in a young female gymnast. Br J Sports Med 2001;35:448-449.
 Sinha AK, Kaeding CC, Wadley GM: Upper extremity stress fractures in athletes: Clinical features of 44 cases. Clin J
Sports Med 1999;9:199-202.
 Caine D, Howe W, Ross W, Bergman G: Does repetitive physical loading inhibit radial growth in female gymnasts? Clin J
Sports Med 1997;7:302-308.
 Chan D, Aldridge MJ, Maffulli N, Davies AM: Chronic stress injuries of the elbow in young gymnasts. Br J Radiol
1991;64:1113-1118.

14. Figure 6 shows the clinical photographs of a newborn who underwent a colostomy
for an imperforate anus. Examination shows extended knees, flexed hips, and
equinovarus feet. Dimpling is noted over the buttocks. Patients with these findings
differ from patients with myelodysplasia in that they

1- have intact motor function.


2- have protective sensation.
3- are at risk for progressive neural deterioration.
4- are at risk for development of a latex allergy.
5- are at risk for development of severe lordosis.

PREFERRED RESPONSE: 2

DISCUSSION: The patient has sacral agenesis. Clinical signs include the classic dimpling over
the buttocks and the characteristic lower extremity deformities. Imperforate anus is often
associated with this disorder. Although motor function correlates with the level of vertebral
defect, sensation is usually intact. This is important therapeutically, because patients are not as
prone to pressure sores as are those with myelodysplasia. Kyphosis may develop in many
patients with lumbosacral agenesis, but lordosis is unusual. Latex allergy and progressive neural
deterioration may occur in patients with either myelodysplasia or sacral agenesis but is more
common in the former.
 Renshaw TS: Sacral agenesis. J Bone Joint Surg Am 1978;60:373-383.

15. Which of the following patients is considered the most appropriate candidate for
selective dorsal rhizotomy?

1- Nonambulatory 2-year-old with spastic diplegia


2- Nonambulatory 2-year-old with spastic quadriplegia
American Academy of Orthopaedic Surgeons
2004 pediatric orthopedic Surgery

3- Nonambulatory 12-year-old with spastic quadriplegia


4- Ambulatory 4-year-old with spastic diplegia
5- Ambulatory 9-year-old with hemiplegia and athetosis

PREFERRED RESPONSE: 4

DISCUSSION: While other surgical and nonsurgical options exist for management of spasticity,
the criteria originally laid out by Peacock and associates describe the most appropriate candidate
for rhizotomy as a patient with spastic diplegia who is between the ages of 4 to 8 years and has a
stable gait pattern that is limited by lower extremity spasticity. Rhizotomy is not recommended
in patients with athetosis because of unpredictable results. In addition, rhizotomy should be
avoided in nonambulatory patients with spastic quadriplegia because it is associated with
significant spinal deformities.
 Peacock WJ, Arens LJ, Berman B: Cerebral palsy spasticity: Selective posterior rhizotomy. Pediatr Neurosci 1987;13:61-
66.
 Oppenheim WL: Selective posterior rhizotomy for spastic cerebral palsy: A review. Clin Orthop 1990;253:20-29.
 Mooney JF III, Millis MB: Spinal deformity after selective dorsal rhizotomy in patients with cerebral palsy. Clin Orthop
1999;364:48-52.

16. A 2-day-old infant has the hyperextended knee deformity shown in Figure 7. No
other deformities are found on examination. A radiograph shows that the ossified
portion of the proximal tibia is slightly anterior to that of the distal femur.
Management should consist of

1- gentle stretching and serial casting.


2- Bryant traction for 1 to 2 weeks, followed by
closed reduction.
3- percutaneous quadriceps recession, followed
by serial casting.
4- delayed open reduction at age 6 months to
avoid iatrogenic damage to either the distal
femoral or proximal tibial physes.
5- a renal ultrasound.

PREFERRED RESPONSE: 1

DISCUSSION: Congenital dislocation of the knee is an uncommon deformity that varies in


presentation from simple hyperextension to complete anterior dislocation of the tibia on the
femur. Treatment varies with the age at presentation and the severity of the deformity. Most
authors recommend early nonsurgical management. A recent study of 24 congenital knee
dislocations in 17 patients found that satisfactory results were obtained in most instances using
American Academy of Orthopaedic Surgeons
2004 pediatric orthopedic Surgery

closed treatment. Based on their findings, the authors concluded that immediate reduction or
serial casting should be performed when the patient is seen early after birth. If the patient is seen
late and correction cannot be achieved by serial casting, traction followed by closed or open
reduction may be necessary. Early percutaneous quadriceps recession has been described for
complex congenital knee dislocations associated with underlying disorders, such as
arthrogryposis and Ehlers-Danlos syndrome.
 Ko JY, Shih CH, Wenger DR: Congenital dislocation of the knee. J Pediatr Orthop 1999;19:252-259.
 Johnson E, Audell R, Oppenheim WL: Congenital dislocation of the knee. J Pediatr Orthop 1987;7:194-200.
 Roy DR, Crawford AH: Percutaneous quadriceps recession: A technique for management of congenital hyperextension
deformities of the knee in the neonate. J Pediatr Orthop 1989;9:717-719.

17. Figures 8a and 8b show the current radiographs of a 10-year-old boy with a hip
disorder who was treated with abduction orthosis 3 years ago. If no further
remodeling occurs, what is the most likely prognosis?

1- The patient is at risk for repeated episodes of ischemic necrosis.


2- The patient is at high risk for deep venous thrombosis.
3- No further problems will develop on the involved side.
4- Accelerated onset of degenerative arthritis will develop on the involved side in the
fifth or sixth decade of life.
5- Epiphyseodesis will be required on the involved side.

PREFERRED RESPONSE:4

DISCUSSION: The radiographs show a child with Legg-Calve-Perthes disease (LCPD) that has
healed. Deformity (asphericity) of the femoral head is evident, but the femoral head and
acetabulum are congruous. Stulberg and associates found that hips with aspherical congruity at
skeletal maturity functioned well until the fifth or sixth decade of life. Similarly, another study
found that degenerative arthritis caused deteriorating hip function after age 40 years in patients
with this degree of residual deformity. Repeated episodes of ischemic necrosis are unlikely.
Although some studies suggested coagulation abnormalities such as protein C and S deficiencies
in children with LCPD, other studies failed to show any evidence of inherited thrombophila in
most children with this disorder. There are no studies to suggest growth acceleration occurs
following LCPD.
American Academy of Orthopaedic Surgeons
2004 pediatric orthopedic Surgery

 Stulberg SD, Cooperman DR, Wallenstein R: The natural history of Legg-Calve-Perthes disease. J Bone Joint Surg Am
1984;66:479-489.
 McAndrew MP, Weinstein SL: A long-term follow-up of Legg-Calve-Perthes disease. J Bone Joint Surg Am 1984;66:860-
869.

18. In girls with idiopathic scoliosis, peak height velocity (PHV) typically occurs at what
point?

1- Before Risser 1 and menarche


2- After Risser 1 and menarche
3- Between Risser 1 and menarche
4- After menarche but before Risser 1
5- At Risser 2

PREFERRED RESPONSE: 1

DISCUSSION: PHV generally occurs while girls are still Risser 0; menarche typically occurs
before Risser 1, which has a wide variation in its timing. The curve magnitude at the PHV is the
best prognostic indicator available. Most untreated patients with curves greater than 30 degrees
at PHV require surgery, while patients with smaller curves at that stage typically do not require
surgery.
 Little DG, Song KM, Katz D, Herring JA: Relationship of peak height velocity to other maturity indicators in idiopathic
scoliosis in girls. J Bone Joint Surg Am 2000;82:685-693.
 Anderson M, Hwang SC, Green WT: Growth of the normal trunk in boys and girls during the second decade of life; related
to age, maturity, and ossification of the iliac epiphyses. J Bone Joint Surg Am 1965;47:1554-1564.

19. Examination of a 6-year-old boy who sustained a displaced Salter-Harris type II


fracture of the distal radius reveals 35 degrees of volar angulation. A satisfactory
reduction is obtained with the aid of a hematoma block. At the 10-day follow-up
examination, radiographs show loss of reduction and 35 degrees of volar angulation.
Management should now consist of

1- Acceptance of the malalignment and continued cast immobilization.


2- Repeat closed reduction with the aid of iv morphine and midazolam.
3- Repeat closed reduction with the aid of iv ketamine
4- Repeat closed reduction with the patient under general anesthesia.
5- Gentle open reduction with smooth cross-pin fixation.

PREFERRED RESPONSE: 1

DISCUSSION: In a 6-year-old child with a physeal fracture, the healing response 10 days after
injury is so advanced that manipulation would have to be very forceful to be successful. A
forceful manipulation in a patient this age increases the risk of early growth arrest and a
significant disability because 80% of the growth of the radius comes from the distal physis.
American Academy of Orthopaedic Surgeons
2004 pediatric orthopedic Surgery

Because of the large contribution of growth from the distal radial physis and the angulation being
in the plane of wrist motion, the potential for remodeling of this fracture is great. It is highly
probable that this fracture will completely remodel in 1 to 2 years of growth. In this patient,
even a "gentle" open reduction would probably require enough force that the physis would be
damaged.
 Dimeglio A: Growth in pediatric orthopaedics, in Morrissy RT, Weinstein SL (eds): Lovell and Winter's Pediatric
Orthopaedics, ed 5. Philadelphia, PA, Lippincott Williams and Wilkins, 2001, pp 33-62.
 Waters PM: Forearm and wrist fractures, in Richards BS (ed): Orthopaedic Knowledge Update: Pediatrics. Rosemont, IL,
American Academy of Orthopaedic Surgeons, 1996, pp 251-258.

20. Figures 9a and 9b show the radiographs of a 12-year-old girl who has had right hip
pain for the past 4 months. She reports that the pain is so severe that she is unable
to walk and is now using a wheelchair. Examination reveals pain with any
attempted range of motion. Management should include

1- Right innominate osteotomy.


2- Right varus upper femoral osteotomy.

3- Open reduction of the right hip.


4- Crutches, physical therapy, and an
abduction brace.
5- MRI of the pelvis.

PREFERRED RESPONSE: 5

DISCUSSION: In addition to mild hip dysplasia, the radiograph shows an osteoblastic lesion of
the right ilium. The patient's symptoms are much more severe than is typical for late hip
dysplasia. MRI can determine the extent of the lesion in the bone and soft tissues. Following
work-up and biopsy, the patient was diagnosed with Ewing's sarcoma.
 Springfield DS, Gebhardt MC: Bone and soft tissue tumors, in Morrissy RT, Weinstein SL (eds): Lovell and Winter's
Pediatric Orthopaedics, ed 5. Philadelphia, PA, Lippincott Williams and Wilkins, 2001, pp 507-518, 542-544.
 Herring JA: Tachdjian's Pediatric Orthopedics, ed 4. Philadelphia, PA, WB Saunders, 2002, p 2030.
American Academy of Orthopaedic Surgeons
2004 pediatric orthopedic Surgery

21. An 18-month-old boy has 45 degrees of kyphosis in the thoracolumbar spine


secondary to type I congenital kyphosis. Examination reveals that he is
neurologically intact, and an MRI scan shows no evidence of intraspinal pathology.
Management should consist of

1- In situ posterior fusion.


2- In situ anterior and posterior fusion.
3- Observation and a follow-up examination in 6 months.
4- Bracing.
5- Anterior decompression and fusion with posterior fusion.

PREFERRED RESPONSE: 1

DISCUSSION: Surgery is indicated for congenital kyphosis once the deformity reaches a certain
size or if significant progression is documented. In a young patient with a relatively small
deformity, the treatment of choice is isolated in situ posterior fusion and postoperative
immobilization. If an adequate posterior fusion can be obtained, an epiphyseodesis effect can be
generated, allowing the remaining anterior growth to cause some correction. Because there is no
evidence of neurologic compression and the deformity is less than 50 degrees, anterior surgery is
not indicated. There is no role for bracing in the management of congenital kyphosis.
 Winter RB: Congenital Deformities of the Spine. New York, NY, Thieme-Stratton, 1983, pp 229-261.
 Winter RB, Moe JH: The results of spinal arthrodesis for congenital spinal deformity in patients younger than five years old.
J Bone Joint Surg Am 1982;64:419-432.

22. Following an acute dislocation of the patella, the risk of a recurrent dislocation is
greater if the patient has which of the following findings?

1- Bipartite patella
2- Age greater than 15 years at the time of the initial dislocation
3- Persistent atrophy of the vastus lateralis
4- Low-lying position of the patella (patella baja)
5- Passive lateral hypermobility of the unaffected knee

PREFERRED RESPONSE: 5

DISCUSSION: Recurrent dislocations may follow an earlier dislocation. One study found that
in patients who had a patellar dislocation between the ages of 11 to 14 years, 60% had a
recurrent dislocation. The incidence of recurrent dislocation dropped to 33% in patients who had
a patellar dislocation between the ages of 15 to 18 years. The authors also found that the
incidence of recurrence was greater in patients who demonstrated a predisposition to dislocation
as determined by evaluation of the unaffected knee. Predisposing signs included passive lateral
hypermobility of the patella, a dysplastic distal third of the vastus medialis obliquis muscle, and
a high and/or lateral position of the patella. A second study found that the risk of redislocation
was considerably higher in patients who were in their teens at the first episode of dislocation
American Academy of Orthopaedic Surgeons
2004 pediatric orthopedic Surgery

compared to older patients. There are no studies linking either a patella baja or a bipartite patella
to an increased risk of redislocation.
 Cash JD, Hughston JC: Treatment of acute patellar dislocation. Am J Sports Med 1988;16:244-249.
 Larsen E, Lauridsen F: Conservative treatment of patellar dislocations: Influence of evident factors on the tendency to
redislocation and the theraputic result. Clin Orthop 1982;171:131-136.

23. Which of the following findings can cause a dorsal bunion in a patient with
neuromuscular disease?

1- Contracted plantar fascia and weakness of the tibialis anterior


2- Overpull of the gastrocnemius-soleus complex and weakness of the anterior
tibialis
3- Overpull of the posterior tibialis and weakness of the peroneals
4- Overpull of the peroneus brevis and weakness of the posterior tibialis
5- Overpull of the tibialis anterior and weakness of the peroneus longus

PREFERRED RESPONSE: 5

DISCUSSION: Unopposed action of the tibialis anterior with weakness of the peroneus longus
will lead to a dorsal bunion and supination deformity. Overpull of the gastrocnemius-soleus
complex and posterior tibialis with weakness of the peroneus brevis will cause equinovarus
deformity. A strong posterior tibialis with weakness of the peroneals will cause varus of the
hindfoot. Unopposed peroneus brevis and incompetence of the posterior tibialis will lead to a
flatfoot deformity. The etiology of a cavus foot is complex, but findings usually include a
contracted plantar fascia and weakness of the tibialis anterior.
 Drennan JC (ed): The Child's Foot and Ankle. New York, NY, Raven Press, 1992, pp 289-302.
 Tachdjian MO: Pediatric Orthopedics ed 2. Philadelphia, PA, WB Saunders, 1990, pp 1937-1946.

24. Which of the following studies is considered most sensitive in monitoring a


therapeutic response in acute hematogenous osteomyelitis?

1- Blood culture
2- WBC count
3- Erythrocyte sedimentation rate (ESR)
4- C-reactive protein
5- Radiography

PREFERRED RESPONSE: 4

DISCUSSION: C-reactive protein declines rapidly as the clinical picture improves. Failure of
the C-reactive protein to decline after 48 to 72 hours of treatment should indicate that treatment
may need to be altered. Blood culture is positive only 50% of the time and will be negative soon
after antibiotics are administered, even if treatment is not progressing satisfactorily. WBC count
American Academy of Orthopaedic Surgeons
2004 pediatric orthopedic Surgery

is highly variable and poorly correlated with treatment. The ESR rises rapidly but declines too
slowly to guide treatment. Radiographic findings may not change but can take up to 2 weeks to
show changes.
 Unidia-Kallio L, et al: Serum c-reactive protein, ESR and WBC in acute hematogenous osteomyelitis in children.
Pediatrics 1994;93:59-62.

25. Figure 10 shows the radiograph of a 7-year-old patient who has a bilateral
Trendelenburg limp and limited range of hip motion but no pain. His work-up
should include

1- a skeletal survey.
2- genetic evaluation.
3- cardiac evaluation.
4- coagulation studies.
5- MRI of the hips.

PREFERRED RESPONSE: 1

DISCUSSION: The radiograph shows bilateral flattening of the femoral heads with mottling and
"fragmentation" suggestive of Legg-Calve-Perthes disease. However, when these changes occur
bilaterally and are symmetric, multiple epiphyseal dysplasia or spondyloepiphyseal dysplasia
should be suspected. Skeletal survey will show irregularity of the secondary ossification centers.
With these conditions, there is no true osteonecrosis and no evidence that orthotic or surgical
"containment" will alter the outcome of progressive degenerative arthritis. Cardiac anomalies
and coagulopathies are not associated with the epiphyseal dysplasias.
 Crossan JF, Wynne-Davies R, Fulford GE: Bilateral failure of the capital femoral epiphysis: Bilateral Perthes disease,
multiple epiphyseal dysplasia, pseudoachondroplasia, and spondyloepiphyseal dysplasia congenita and tarda. J Pediatr
Orthop 1983;3:297-301.
 Sponseller PD: The skeletal dysplasias, in Morrissy RT, Weinstein SL (eds): Lovell and Winter's Pediatric Orthopaedics, ed
5. Philadelphia, PA, Lippincott Williams and Wilkins, 2001, pp 269-270.

26. Figures 11a and 11b show the clinical photograph and radiograph of a newborn.
Based on these findings, what is the best course of action?

1- Genetic testing
2- Stretching exercises performed by the parents
3- A series of corrective plaster casts
4- A long leg orthosis to control the foot and leg
American Academy of Orthopaedic Surgeons
2004 pediatric orthopedic Surgery

5- Surgical lengthening of the anterior tibial


tendon and a short leg cast

PREFERRED RESPONSE: 2

DISCUSSION: The newborn has posteromedial bowing of the tibia and calcaneal valgus
deformity of the foot. Both are thought to be caused by abnormal intrauterine positioning. The
foot deformity typically responds to stretching. The tibial bowing straightens with growth. The
long-term problem is limb-length discrepancy.
 Heyman CH, Herndon CH, Heiple KG: Congenital posterior angulation of the tibia with talipes calcaneus. J Bone Joint
Surg Am 1959;41:476-488.
 Hofmann A, Wenger DR: Posteromedial bowing of the tibia: Progression of discrepancy in leg lengths. J Bone Joint Surg
Am 1981;63:384-388.

27. Figure 12 shows the radiograph of a patient who has anterior


knee pain. History reveals a femoral fracture at age 5 years.
What is the most likely cause of the deformity?

1- Osgood-Schlatter disease
2- Patellar tendon rupture
3- Posterior cruciate ligament rupture
4- Overlengthened hamstrings 5-
5- Tibial tubercle growth arrest

PREFERRED RESPONSE: 5

DISCUSSION: The radiograph shows a recurvatum deformity of the proximal tibia with growth
arrest of the tibial tubercle apophysis. This deformity has been described in association with
femoral shaft fractures in children and has been attributed to a clinically silent, concommitant
American Academy of Orthopaedic Surgeons
2004 pediatric orthopedic Surgery

injury to the proximal tibial physes and also to iatrogenic injury associated with a proximal tibial
traction pin. Overlengthened hamstrings and rupture of the posterior cruciate ligament may lead
to knee hyperextension; however, these problems should not cause bone deformity. Osgood-
Schlatter disease occurs when growth is nearly complete and usually leads to prominence of the
tibial tubercle. Patellar tendon rupture is rare in children and would not cause this deformity
unless the repair was performed with screws across the apophysis.
 Hresko MT, Kasser JR: Physeal arrest about the knee associated with non-physeal injuries of the lower extremity. J Bone
Joint Surg Am 1989;71:698-703.
 Bowler JR, Mubarak SJ, Wenger DR: Tibial physeal closure and genu recurvatum after femoral fracture: Occurrence
without a tibial traction pin. J Pediatric Orthop 1990;10:653-657.

28. An 11-year-old boy has had a fever and pain and swelling over the lateral aspect of
his right ankle for the past 3 days. Examination reveals warmth, swelling, and
tenderness over the lateral malleolus, and he has a temperature of 103.2 degrees F
(39.5 degrees C). Laboratory studies show a WBC count of 13,200/mm3 with 61%
neutrophils, an erythocyte sedimentation rate of 112 mm/h, and a C-reactive protein
of 15.7. Radiographs and a T2-weighted MRI scan are shown in Figures 13a
through 13c. Aspiration yields 1 mL of purulent fluid. Management should now
consist of

1- Oral antibiotics and a follow-up office appointment the next day.


2- Incision and drainage of the distal fibular metaphysis.
3- Indium-labeled wbc scan.
4- Antituberculous medication for 6 months.
5- Three-phase technetium Tc 99m bone scan.

PREFERRED RESPONSE: 2

DISCUSSION: The initial signs and symptoms of acute hematogenous osteomyelitis vary widely
but usually include fever, bone pain, and impaired use of the involved extremity. In lower
extremity infections, the child may limp or refuse to walk. Examination often reveals bone
tenderness. In more advanced cases, erythema, warmth, and swelling may be present. The WBC
American Academy of Orthopaedic Surgeons
2004 pediatric orthopedic Surgery

and neutrophil counts are not always elevated, but the erythocyte sedimentation rate will be
abnormal in more than 90% of patients. When the infection is diagnosed early, before a
subperiosteal abscess has formed, antibiotics alone may be adequate to treat the infection. This
patient has a more advanced infection, however, with the MRI scan revealing a subperiosteal
abscess that was confirmed by aspiration. When an abscess is present, surgical drainage is
generally indicated to remove devitalized tissue and to enhance the efficacy of the antibiotics.
Further studies, such as bone or indium scans, are not necessary and will delay definitive
treatment.
 Scott RJ, Christofersen MR, Robertson WW Jr, et al: Acute osteomyelitis in children: A review of 116 cases. J Pediatr
Orthop 1990;10:649-652.
 Vaughan PA, Newman NM, Rosman MA: Acute hematogenous osteomyelitis. J Pediatr Orthop 1987;7:652-655.

29. Figure 14 shows the clinical photographs and radiograph of an 8-year-old girl who
has a progressive equinus deformity of the right ankle. There is no history of
trauma or infection. What is the most likely diagnosis?

1- Arthrogryposis
2- Melorheostosis
3- Focal scleroderma
4- Calcaneal osteomyelitis
5- Klippel-Trenaunay-Weber syndrome

PREFERRED RESPONSE: 3

DISCUSSION: Focal scleroderma is characterized by the formation of patches of sclerotic skin,


also known as morphea, or streaks of sclerosis (linear scleroderma). Systemic involvement in
focal scleroderma is unusual; however, progression during childhood is common. Contracture of
underlying tissues is common, often resulting in serious joint contractures. Bony changes similar
to those seen in melorheostosis can be seen. This patient has characteristic skin changes, atrophy
of the soft tissues, Achilles tendon contractures, and calcaneal deformities. There are no signs of
arthrogryposis, which usually presents with bilateral congenital deformities, including
equinovarus. Klippel-Trenaunay-Weber syndrome is characterized by venous malformation in
association with focal overgrowth.
 Bottoni CR, Reinker KA, Gardner RD, Person DA: Scleroderma in childhood: A 35-year history of cases and review of the
literature. J Pediatr Orthop 2000;20:442-449.
American Academy of Orthopaedic Surgeons
2004 pediatric orthopedic Surgery

30. Which of the following patients is considered the most appropriate candidate for an
isolated split posterior tendon transfer?

1- A 2-year-old with spastic diplegia and a flexible equinovarus foot


2- A 5-year-old with spastic quadriplegia and athetosis
3- A 6-year-old with spastic hemiplegia, a flexible equinovarus foot, and out-of-
phase posterior tendon activity on electromyography
4- A 6-year-old with spastic hemiplegia, a flexible equinovarus foot, and continuous
posterior tibial tendon activity on electromyography
5- A 10-year-old with spastic quadriplegia and a rigid equinovarus foot

PREFERRED RESPONSE: 4

DISCUSSION: Isolated split posterior tendon transfer alone is best performed in a patient with
cerebral palsy who is between the ages of 4 and 7 years and has a flexible equinovarus foot.
Rigid deformities often must be managed with a combination of soft-tissue and bony procedures.
Patients with out-of-phase activity may be best managed with a transfer of the posterior tibialis
to the dorsum of the foot, while those with continuous activity are better candidates for an
isolated split posterior tendon transfer.
 Drennan JC (ed): The Child's Foot and Ankle. New York, NY, Raven Press, 1992, pp 291-294.
 Green NE, Griffin PP, Shiavi R: Split posterior tibial-tendon transfer in cerebral palsy. J Bone Joint Surg Am 1983;65:748-
754.
 Kling TF Jr, Kaufer H, Hensinger RN: Split posterior tibial-tendon transfers in children with cerebral spastic paralysis and
equinovarus deformity. J Bone Joint Surg Am 1985;67:186-194.

31. The mother of a 2-year-old boy reports that he had pain in the right hip all night
and refuses to walk on the leg this morning. He is afebrile. Examination reveals
pain on hip extension and adduction, but he is able to internally and externally
rotate the hip approximately 20 degrees in each direction without pain. Laboratory
studies reveal a WBC count of 7,400/mm3, with 62% polymorphonuclear neutrophil
leukocytes. The AP radiograph shown in Figure 15 reveals a left teardrop distance
of 8 mm, while the right side
measures 10 mm. Which of the
following diagnostic studies will
best help confirm the diagnosis?

1- Technetium Tc 99m bone scan


2- Ultrasound of the hips
3- CT of the pelvis
4- MRI of the pelvis
5- Aspiration of the hip joint

PREFERRED RESPONSE: 5
American Academy of Orthopaedic Surgeons
2004 pediatric orthopedic Surgery

DISCUSSION: The differential diagnosis includes septic hip and transient synovitis. Both
disorders may present with a joint effusion. The increased teardrop distance and loss of range of
motion implies that there is excess fluid in the joint. While the other tests can confirm this, only
aspiration can characterize the fluid further, thereby indicating the etiology of the effusion.
Provided there is enough fluid obtained at aspiration, the joint fluid should be sent for Gram
stain, culture, cell count, glucose, and protein studies.
 Tachdjian MO: Pediatric Orthopedics, ed 2. Philadelphia, PA, WB Saunders, 1990, pp 1415-1435.

32. A senior resident is scheduled to perform a posterior medial release on a 10-month-


old infant who has a congenital clubfoot deformity. Informed consent is obtained
for the procedure. The supervising surgeon is obligated to give the parents what
information?

1- The assistant is a senior resident and he or she may perform nonessential parts of
the procedure.
2- The resident will be performing the procedure and the supervising surgeon will
actively participate.
3- The supervising surgeon will be doing the procedure to allay the parents' anxiety.
4- No information needs to be given about who will be performing the procedure
because it is implied that the residents perform all procedures at a teaching
hospital.
5- No information needs to be given about who will be performing the procedure
unless the parents ask.

PREFERRED RESPONSE: 2

DISCUSSION: Informed consent is generally considered to be a process of mutual decision


making between the physician and patient. The physician is required to provide to the patient all
material information that is needed for the patient to make an informed decision. The courts
have held that a patient's choice of surgeon is as important to the consent as the procedure itself.
Assistance by a surgical trainee with adequate supervision is permissible when there has been
adequate disclosure. Adequate supervision may be defined as active participation by the
attending during the essential parts of the procedure. Allowing a substitute surgeon to operate on
a patient without the patient's knowledge "ghost surgery" may result in charges of battery against
the substitute surgeon and malpractice against the surgeon to whom the patient gave consent.
 Kocher MS: Ghost surgery: The ethical and legal implications of who does the operation. J Bone Joint Surg Am
2002;84:148-150.
 Holmes MK: Ghost surgery. Bull NY Acad Med 1980;56:412-419.

33. Figure 16 shows the clinical photograph of a 3-month-old infant with a foot
deformity that has been nonprogressive since birth. Examination reveals that the
deformity corrects actively and with passive manipulation. There is no associated
equinus. Management should consist of
American Academy of Orthopaedic Surgeons
2004 pediatric orthopedic Surgery

1- serial casting.
2- UCBL orthotics.
3- abductor hallucis lengthening.
4- observation and parental
reassurance.
5- corrective shoes.

PREFERRED RESPONSE: 4

DISCUSSION: The patient has bilateral metatarsus adductus deformities. In a long-term follow-
up study by Farsetti and associates, deformities that were passively correctable spontaneously
resolved and no treatment was required. More rigid deformities were successfully treated with
serial manipulation, with good results in 90%. There were no poor results. Therefore,
observation is the management of choice for passively correctable deformities. In feet that are
more rigid, serial manipulation and casting is the management of choice.
 Farsetti P, Weinstein SL, Ponseti IV: The long-term functional and radiographic outcomes of untreated and non-operatively
treated metatarsus adductus. J Bone Joint Surg Am 1994;76:257-265.

34. Figure 17 shows the radiograph of an 11-year-old boy with Duchenne muscular
dystrophy who has been nonambulatory for the past 2 years. Management of the
spinal deformity should consist of

1- Wheelchair modifications and custom-molded


inserts.
2- Posterior fusion with instrumentation.
3- Anterior and posterior fusion.
4- Observation and reexamination in 6 months.
5- Thoracolumbosacral orthosis bracing.

PREFERRED RESPONSE: 2

DISCUSSION: The presence of any curve greater than 20 degrees in a nonambulatory patient
with Duchenne muscular dystrophy is an indication for posterior fusion with instrumentation.
Because of progressive cardiomyopathy and pulmonary deficiency, waiting until the curve is
larger can increase the risk of pulmonary or cardiac complications during or following surgery.
American Academy of Orthopaedic Surgeons
2004 pediatric orthopedic Surgery

There is some disagreement as to whether all such fusions must extend to the pelvis. Bracing or
other nonsurgical management is ineffective and is not indicated in this situation.
 Sussman M: Duchenne muscular dystrophy. J Am Acad Orthop Surg 2002;10:138-151.
 Mubarek SJ, Morin WD, Leach J: Spinal fusion in Duchenne muscular dystrophy: Fixation and fusion to the sacropelvis? J
Pediatr Orthop 1993;13:752-757.

35. A 13-year-old girl with Down syndrome has bilateral chronic patellar dislocations.
She denies knee pain. She is able to straighten her knees and walks with a
symmetric but awkward gait. She does not flex her knees in midstance.
Examination reveals that the patellae cannot be brought into a reduced position.
Management should consist of

1- Lateral retinacular release and medial reefing.


2- Patellar tendon transfer medially.
3- Lateral release and patellar tendon transfer.
4- Femoral and tibial derotation osteotomies.
5- Continued observation.

PREFERRED RESPONSE: 5

DISCUSSION: Chronic dislocation of the patella is occasionally seen in patients with Down
syndrome. In early childhood, patellar realignment may restore stability of the patellae. In later
childhood, bony changes in the patellar groove interfere with stability, even if surgical
realignment is performed. Realignment can also lead to increased knee pain postoperatively. In
asymptomatic patients who are able to extend their knees, continued observation is the
management of choice.
 Dugdale TW, Renshaw TS: Instability of the patellofemoral joint in Down syndrome. J Bone Joint Surg Am 1986;68:405-
413.
 Mendez AA, Keret D, MacEwen GD: Treatment of patellofemoral instability in Down's syndrome. Clin Orthop
1988;234:148-158.

36. A 3-year-old patient with L3 myelomeningocele has bilateral dislocated hips.


Management should consist of

1- observation.
2- bilateral open reduction.
3- bilateral open reduction and psoas transfers.
4- bilateral open reduction and external oblique transfers.
5- bilateral valgus osteotomies.

PREFERRED RESPONSE: 1

DISCUSSION:In patients with myelomeningocele, the presence of bilateral hip dislocation does
not affect ambulation, bracing requirements, sitting ability, degree of scoliosis, or level of
American Academy of Orthopaedic Surgeons
2004 pediatric orthopedic Surgery

comfort. There is little evidence to support active treatment of bilateral hip dislocations in
patients with myelomeningocele proximal to L4.
 Fraser RK, Hoffman EB, Sparks LT, et al: The unstable hip and mid-lumbar myelomeningocele. J Bone Joint Surg Br
1992;74:143-146.
 Richards BS (ed): Orthopaedic Knowledge Update: Pediatrics. Rosemont, IL, American Academy of Orthopaedic
Surgeons, 1996, pp 65-76.

37. A 4-year-old child sustains a spiral fracture to the tibia in an unwitnessed fall.
History reveals three other fractures to long bones, and the parents are vague about
the etiology of each. There is no family history of bone disease. The parents ask if
the child has osteogenesis imperfecta (OI); however, there are no clinical or
radiographic indications of this diagnosis. In addition to fracture care, management
should include

1- Notification of child protective services and hospital admission.


2- A punch biopsy of skin for collagen analysis.
3- DNA testing for OI.
4- Calcium, phosphate, and alkaline phosphatase studies.
5- Placement of intramedullary rods to prevent further fractures.

PREFERRED RESPONSE: 1

DISCUSSION: Child abuse and OI are frequently both in the differential diagnosis of a child
with multiple fractures. If OI is suspected, testing is appropriate to confirm this diagnosis. This
may include skull radiographs to look for wormian bones and/or fibroblast culturing and
collagen analysis of a punch biopsy. Unfortunately, because of the large number of mutations
that can yield the disease, DNA testing is not commercially available for OI. In this patient,
however, the physician suspects nonaccidental trauma and is legally obligated in most states to
notify child protective services. Because the child may be at considerable risk of further injury,
hospitalization is indicated to protect the child until child protective services can complete a
home investigation and assess the degree of risk. Work-up for both OI and child abuse can be
done during the hospitalization.
 Rockwood CA, Wilkins KE, King RE (eds): Fractures in Children. Philadelphia, PA, JB Lippincott, 1984, vol 3, pp 173-
175.
 Kempe CH, Silverman FN, Stelle BF, Droegemueller W, Silver HK: The battered-child syndrome. JAMA 1962;181:17-24.
 Akbarnia BA, Akbarnia NO: The role of the orthopedist in child abuse and neglect. Orthop Clin North Am 1976;7:733-742.

38. A 10-year-old girl has been unable to walk for the past 5 days because of bilateral
hip pain. Administration of IV morphine has provided some pain relief. She is
afebrile. History reveals that she had an upper respiratory tract infection 3 weeks
ago that resolved uneventfully. Examination reveals moderate pain with internal
rotation and abduction, while log rolling maneuvers do not cause significant pain.
An MRI scan shows a small effusion of one hip; however, a bone scan and plain
American Academy of Orthopaedic Surgeons
2004 pediatric orthopedic Surgery

radiographs are normal. Initial laboratory studies showed a markedly elevated


WBC count, which subsequently declined to normal levels with IV antibiotics.
Current studies show an erythrocyte sedimentation rate (ESR) of 100 mm/h.
Aspiration of the hip obtains 3 mL of fluid; Gram stain is negative for bacteria, but
a cell count shows a WBC count of 16,500/mm3. Streptozyme titer of the peripheral
blood is 200 units (normal is less than 100 units). Management should now consist
of

1- Discontinuation of antibiotics and administration of nonsteroidal anti-


inflammatory drugs, followed by physical therapy.
2- Incision and drainage of both hips.
3- In situ pinning of both hips.
4- Broad-spectrum antibiotics in accordance with the recommendations of a
pediatric infectious diseases consultant.
5- Continued use of IV antibiotics for 2 weeks, followed by oral antibiotics until the
ESR returns to normal.

PREFERRED RESPONSE: 1

DISCUSSION: This clinical situation requires careful analysis because some data suggest
infection and some a noninfectious inflammatory process. Bilateral hip involvement, the
absence of significant fluid collection or fever, the streptozyme level, and the history of upper
respiratory infection all suggest poststreptococcal toxic synovitis as the most likely cause for the
clinical presentation. In the first 24 hours, this type of presentation might warrant incision and
drainage given uncertainty of the diagnosis and the risks associated with missing an infection.
However, 5 days after onset, surgery is not warranted, especially given that the patient remains
afebrile and her symptoms are improving. Cardiology consultation would be appropriate. There
is no evidence to suggest slipped capital femoral epiphysis. Treatment with antibiotics is not
advised because there is no bacteriologic data on which to base treatment.
 De Cunto CL, Giannini EH, Fink CW, et al: Prognosis of children with poststreptococcal reactive arthritis. J Pediatr Infect
Dis 1988;7:683-686.
 Haueisen DC, Weiner DS, Weiner SD: The characterization of "transient synovitis of the hip" in children. J Pediatr Orthop
1986;6:11-17.

39. What is the best initial screening test for a patient with a limb-length discrepancy?

1- CT scenography
2- Orthoroentgenography (scanography)
3- Measurement from the anterior superior iliac spine to the malleolus with a tape
measure
4- With the patient standing, placement of blocks under the short leg to level the
pelvis, followed by measurement of the blocks
5- Galeazzi test
American Academy of Orthopaedic Surgeons
2004 pediatric orthopedic Surgery

PREFERRED RESPONSE: 4

DISCUSSION: With the patient standing, add blocks under the short leg until the pelvis is level,
then measure the blocks to determine the discrepancy. This method is an accurate, simple, and
inexpensive way to assess limb-length discrepancy. Differences of less than 2 cm need no
treatment. Increasing discrepancy in a growing child should be followed clinically.
Radiographic examination can include scanography, CT scanography, or a standing pelvic
radiograph with the pelvis leveled. CT scanography is the most accurate diagnostic test when
hip, knee, or ankle contractures are present.
 Herring JA: Tachdjian's Pediatric Orthopedics, ed 4. Philadelphia, PA, WB Saunders, 2002, pp 1041-1045.
 Schoenecker PL, Rich MM: The lower extremity, in Morrissy RT, Weinstein SL (eds): Lovell and Winter's Pediatric
Orthopaedics, ed 5. Philadelphia, PA, Lippincott Williams and Wilkins, 2001, pp 1120-1122.
 Stanitski DF: Limb-length inequality: Assessment and treatment options. J Am Acad Orthop Surg 1999;7:143-153.
 Aaron A, Weinstein D, Thickman D, Eilert R: Comparison of orthoroentgenography and computed tomography in the
measurement of limb-length discrepancy. J Bone Joint Surg Am 1992;74:897-902.

40. A 6-year-old girl has never been able to crawl or walk and can sit only when
propped. History reveals no complications during pregnancy or delivery.
Examination reveals a 30-degree scoliosis from T4 to L3. Deep tendon reflexes are
absent, but fasciculations are present. The most likely genetic defect is the result of
an abnormality in

1- Peripheral myelin protein 22.


2- Connexin 32.
3- Survival motor neuron.
4- Neurofibromin.
5- Frataxin.

PREFERRED RESPONSE: 3

DISCUSSION: The patient's findings are consistent with an intermediate form of spinal
muscular atrophy. Children with this condition appear normal at birth but are not able to walk.
The disorder affects anterior horn cells. Fasciculations may be present, but deep tendon reflexes
are typically absent. The development of scoliosis is almost universal with this type of spinal
muscular atrophy. More than 90% of patients with spinal muscular atrophy have deletions in the
telomeric survival motor neuron gene. Peripheral myelin protein 22 is abnormal in Charcot-
Marie-Tooth type IA. Connexin 32 is abnormal in the X-linked type of Charcot-Marie-Tooth
disease. Neurofibromin is affected in neurofibromatosis type 1. Friedreich's ataxia is secondary
to a disorder of frataxin.
 Buckwalter JA, Einhorn TA, Simon SR (eds): Orthopaedic Basic Science, ed 2. Rosemont, IL, American Academy of
Orthopaedic Surgeons, 2000, pp 111-131.
 Thompson GH, Berenson FR: Other neuromuscular disorders, in Morrissy RT, Weinstein SL (eds): Lovell and Winter's
Pediatric Orthopaedics, ed 5. Philadelphia, PA, Lippincott Williams and Wilkins, 2001, pp 633-676.
American Academy of Orthopaedic Surgeons
2004 pediatric orthopedic Surgery

41. Figure 18a shows the clinical photograph of a 2-year old boy who has a deformity of
the right leg. Examination reveals eight cutaneous markings similar to those shown
in Figure 18b. Radiographs are shown in Figure 18c. Management should consist
of

1- Fragmentation, realignment, and intramedullary nailing of the tibia.


2- Resection of the dysplastic region of the tibia and insertion of a vascularized
fibula.
3- Supplemental vitamin D and phosphate.
4- A clamshell orthosis.
5- Observation for spontaneous remodeling.

PREFERRED RESPONSE: 4

DISCUSSION: The diagnosis of neurofibromatosis may be based on the presence of at least six
cafe-au-lait spots larger than 5 mm in diameter and the osseous lesion shown in Figure 18c.
Neurofibromatosis occurs in 50% of patients who have an anterolateral bowing deformity of the
tibia, and this bowing may be the first clinical manifestation of this disorder. The patient has
anterolateral bowing of the tibia and fibula that warrants concern for a possible fracture and
pseudarthrosis; therefore, the limb should be protected in a total contact orthosis to prevent
fracture. In contradistinction to posteromedial bowing of the tibia and fibula, spontaneous
remodeling of an anterolateral bowing deformity is not expected. Intramedullary nailing or the
use of a vascularized fibula is reserved for the treatment of a congenital pseudarthrosis of the
tibia.
 Crawford AH Jr, Bagamery N: Osseous manifestations of neurofibromatosis in childhood. J Pediatr Orthop 1986;6:72-88.
 Schoenecker PL, Rich MM: The lower extremity, in Morrissy RT, Weinstein SL (eds): Lovell and Winter's Pediatric
Orthopaedics, ed 5. Philadelphia, PA, Lippincott Williams and Wilkins, 2001, vol 2, pp 1059-1104.

42. What is the most common problem seen following epiphysiodesis for limb-length
discrepancy?

1- Fracture through the site of the physeal resection


American Academy of Orthopaedic Surgeons
2004 pediatric orthopedic Surgery

2- Direct neurovascular injury


3- Incomplete growth arrest from inadequate physeal excision
4- Persistent discrepancy from an error in the timing of the surgery
5- Cartilaginous injury because of inadvertent joint penetration

PREFERRED RESPONSE: 4

DISCUSSION: Errors in timing are by far the most common in this technically safe procedure.
Incomplete growth arrest has been reported in up to 15% of patients versus timing errors in 61%.
Fracture through the site has been reported rarely. Neurovascular and cartilaginous injury are
extremely uncommon but always need to be considered when performing surgery in the vicinity
of these structures.
 Blair VP III, Walker SJ, Sheridan JJ, Schoenecker PL: Epiphysiodesis: A problem of timing. J Pediatr Orthop 1982;2:281-
284.
 Raney ER: Limb-length discrepancy, in Fitzgerald RH, Kaufer H, Malkani AL (eds): Orthopaedics. St Louis, MO, Mosby,
2002, pp 1519-1526.

43. Figure 19 shows the radiograph of a 6-month-old infant who has limited hip motion.
History reveals no complications during pregnancy or delivery. Examination
reveals that hip abduction is 45 degrees in flexion bilaterally. The neurologic
examination is normal. What is the best course of action?

1- Adductor tenotomy
2- Physical therapy
3- Observation
4- Abduction orthosis
5- Neurologic consultation

PREFERRED RESPONSE: 3

DISCUSSION: Diminished hip abduction can occur in normal children and is not always
associated with hip pathology; therefore, initial management should consist of observation.
 Castelein RM, Korte J: Limited hip abduction in the infant. J Pediatr Orthop 2001;21:668-670.

44. A 6-year-old boy with spastic diplegic cerebral palsy has a crouched gait.
Examination reveals hip flexion contractures of 15 degrees and popliteal angles of
70 degrees. Equinus contractures measure 10 degrees with the knees extended.
American Academy of Orthopaedic Surgeons
2004 pediatric orthopedic Surgery

Which of the following surgical procedures, if performed alone, will worsen the
crouching?

1- Iliopsoas release from the lesser trochanter


2- Iliopsoas release at the pelvic brim
3- Hamstring lengthening
4- Heel cord lengthening
5- Split posterior tibial tendon transfer

PREFERRED RESPONSE: 4

DISCUSSION: Children with spastic diplegic cerebral palsy often have contractures of multiple
joints. Because the gait abnormalities can be complex, isolated surgery is rarely indicated. To
avoid compensatory deformities at other joints, it is preferable to correct all deformities in a
single operation. Isolated heel cord lengthening in the presence of tight hamstrings and hip
flexors will lead to progressive flexion at the hips and knees, thus worsening a crouched gait.
Split posterior tibial tendon transfer is used for patients with hindfoot varus, which is not present
in this patient.
 Gage JR: Distal hamstring lengthening/release and rectus femoris transfer, in Sussman MD (ed): The Diplegic Child.
Rosemont, IL, American Academy of Orthopaedic Surgeons, 1992, pp 324-326.
 Bleck EE: Orthopaedic Management of Cerebral Palsy, in Saunders Monographs in Clinical Orthopaedics. Philadelphia, PA,
WB Saunders, vol 2, 1979.

45. The parents of a 13-year-old boy with


Down syndrome report that he has an
increasing limp and decreased
endurance with activities. Lateral
flexion-extension radiographs of the
cervical spine show no evidence of
instability. Examination reveals a
right Trendelenburg limp and an
obvious limb-length discrepancy. Hip
motion is symmetric except for some
decreased abduction on the right side.
A standing AP radiograph is shown in
Figure 20. Management should
consist of

1- Observation.
2- A shoe lift.
3- Abduction bracing.
4- Nonsteroidal anti-inflammatory drugs.
5- Capsulorrhaphy and pelvic and femoral osteotomies.

PREFERRED RESPONSE: 5
American Academy of Orthopaedic Surgeons
2004 pediatric orthopedic Surgery

DISCUSSION: Ligamentous laxity and muscle hypotonia seen in Down syndrome contribute to
the incidence of hip subluxation and dislocation. These factors can be progressive and lead to
degenerative arthritis in adults with Down syndrome. Because this patient has a progressive
limp and decreased endurance, observation and a shoe lift are not options. Bracing may be an
option in the younger child before significant bony changes occur. Surgical intervention is the
treatment of choice in this patient; however, all components of the deformity need to be
addressed. Because of the increased capsular laxity, there is a high likelihood of recurrence if
capsulorrhaphy is not included with the pelvic and femoral osteotomies. Surgery in these
patients is associated with a high rate of complications.
 Shaw ED, Beals RK: The hip joint in Down's syndrome: A study of its structure and associated disease. Clin Orthop
1992;278:101-107.
 Aprin H, Zinc WP, Hall JE: Management of dislocation of the hip in Down's syndrome. J Pediatr Orthop 1985;5:428-431.
 Greene WB: Closed treatment of hip dislocation in Down syndrome. J Pediatr Orthop 1998;18:643-647.

46. A 9-year-old girl reports the immediate onset of severe groin pain and the inability
to walk after tripping on a curb. Examination reveals marked hip pain with passive
range of motion. A radiograph is shown in Figure 21. Regardless of treatment,
what is the most common complication following this injury?

1- Chondrolysis
2- Osteochondritis dissecans of
the femoral head
3- Osteonecrosis of the femoral
head
4- Nonunion
5- Coxa magna

PREFERRED RESPONSE: 3

DISCUSSION: The patient has an unstable slipped capital femoral epiphysis (SCFE).
According to the classification system based on physeal stability, an unstable SCFE is one in
which the patient is unable to walk, even with crutches. Ishemic necrosis, or osteonecrosis, of
the femoral head is the most devastating complication of SCFE. One study found a 47%
incidence of ischemic necrosis following unstable slips. This complication is most likely the
result of vascular injury associated with initial femoral head displacement rather than the result
of either tamponade from joint effusion or gentle repositioning prior to stabilization.
Chondrolysis is a relatively uncommon complication following treatment of SCFE. This
complication has been associated with persistent penetration of the hip joint with screws or pins
used to stabilize the femoral head or with spica cast immobilization. There are no reports to
suggest that osteochondritis dissecans, nonunion, or coxa magna follows treatment of SCFE.
American Academy of Orthopaedic Surgeons
2004 pediatric orthopedic Surgery

 Loder RT, Richards BS, Shapiro PS, et al: Acute slipped capital epiphysis: The importance of physeal stability. J Bone Joint
Surg Am 1993;75:1134-1140.
 Rhoad RC, Davidson RS, Heyman S, et al: Pretreatment bone scan in SCFE: A predictor of ischemia and avascular necrosis.
J Pediatr Orthop 1999;19:164-168.

47. An 8-year-old boy reports progressive difficulty with walking. Examination reveals
muscle weakness, with proximal groups more affected than distal muscles. Deep
tendon reflexes are within normal limits. Laboratory studies show a creatine kinase
level of 7,200 IU. Based on these findings, what is the most likely diagnosis?

1- Becker muscular dystrophy


2- Spinal muscular atrophy, type III
3- Emery-Dreifuss dystrophy
4- Limb girdle dystrophy
5- Guillain-Barre syndrome

PREFERRED RESPONSE: 1

DISCUSSION: Patients with Becker muscular dystrophy have an abnormality in dystrophin, but
unlike patients with Duchenne muscular dystrophy, some dystrophin is present. As a result, the
progression of muscle weakness is slower, with the diagnosis typically made after age 8 years.
Similar to patients with Duchenne muscular dystrophy, patients with Becker muscular dystrophy
have pseudohypertrophy of the calves, markedly increased creatine kinase levels, and X-linked
transmission of the condition. In addition, these patients are more prone to cardiomyopathy; a
condition that should be carefully evaluated if any surgery is required. Patients with spinal
muscular atrophy also have proximal muscle weakness, but the onset of weakness occurs earlier
in childhood. These patients also have absent deep tendon reflexes and fasciculations, but
pseudohypertrophy is absent and creatine kinase levels are normal. Patients with Emery-
Dreifuss dystrophy may have a similar clinical picture to Becker's muscular dystrophy, but
pseudohypertrophy is absent and creatine kinase levels are only mildly elevated. In addition,
neck extension, elbow flexion, and ankle equinus contractures develop at an early age. Limb
girdle dystrophy is a group of progressive muscular dystrophies that is not associated with
pseudohypertrophy or a significant elevation of creatine kinase levels. Guillain-Barre syndrome
is a condition associated with results from postinfectious demyelination of the peripheral nerve.
These patients have the acute onset of weakness, hypotonia, and areflexia; creatine kinase levels
are normal.
 Sussman MD: Muscular dystrophy, in Fitzgerald RH, Kaufer H, Malkani AL (eds): Orthopaedics. St Louis, MO, Mosby,
2002, pp 1573-1583.
 Thompson GH, Berenson FR: Other neuromuscular disorders, in Morrissy RT, Weinstein SL (eds): Lovell and Winter's
Pediatric Orthopaedics, ed 5. Philadelphia, PA, Lippincott Williams and Wilkins, 2001, vol 1, pp 633-676.

48. A 7-year-old boy sustained a 2-cm laceration to the anterior aspect of his left knee
after falling on a rock. Examination reveals that the joint surface is not visible
American Academy of Orthopaedic Surgeons
2004 pediatric orthopedic Surgery

through the wound. Radiographs show no evidence of a foreign body or free air in
the joint. Management should consist of

1- Debridement of the skin edges, closure of the wound, and administration of an


oral cephalosporin for 10 days.
2- A saline load test.
3- Arthroscopic examination of the knee.
4- Leaving the wound open, administration of an oral cephalosporin, and
reevaluation in 48 hours.
5- Injection of contrast material into the laceration, followed by radiographic studies.

PREFERRED RESPONSE: 2

DISCUSSION: The possibility of an open joint injury should be considered in any patient who
has a small periarticular laceration. Failure to promptly diagnose and treat such injuries may
lead to septic arthritis. The diagnosis of an open joint is easily made when there is visible
communication of the joint through the traumatic wound, or when intra-articular air is present on
a radiograph. In the absence of these findings, the diagnosis of an open joint may be established
by the saline load test, in which a volume of saline is injected into the joint under sterile
conditions. If fluid extravasates through the traumatic wound, the diagnosis of an open joint is
established. Voit and associates used a saline load test in 50 patients with periarticular
lacerations suggestive of joint penetration. When they compared the clinical prediction of
whether or not the laceration had penetrated the joint and the test results, the authors reported a
false-positive clinical result in 39% of patients and a false-negative clinical result in 43%. The
authors concluded that the saline load test was valuable in evaluating periarticular lacerations.
 Voit GA, Irvine G, Beals RK: Saline load test for penetration of periarticular lacerations. J Bone Joint Surg Br
1996;78:732-733.
 Leffers D: Dislocations and soft tissue injuries of the knee, in Browner BD (ed): Skeletal Trauma, ed 1. Philadelphia, PA,
WB Saunders, 1992, pp 1717-1743.

49. What radiographic measurement is best used to assess the adequacy of deformity
correction for the patient shown in Figure 22?

1- Reimers index (migration


index)
2- Acetabular index
3- Center edge angle of Wiberg
4- Hilgenreiner physeal angle
5- Head-shaft angle (Southwick
angle)

PREFERRED RESPONSE: 4
American Academy of Orthopaedic Surgeons
2004 pediatric orthopedic Surgery

DISCUSSION: Developmental coxa vara develops in early childhood and results in a


progressive decrease in the proximal femoral neck-shaft angle with growth. The characteristic
radiographic features are seen in this patient and include a decreased neck-shaft angle, a more
vertical position of the physeal plate, and a triangular metaphyseal fragment in the inferior
femoral neck, surrounded by an inverted radiolucent Y pattern. The main goal of surgery is to
correct the varus angulation into a more normal range. Valgus overcorrection is preferred. A
recent study emphasized the importance of adequately correcting the Hilgenreiner physeal angle
to less than 38 degrees to minimize the risk of recurrent angulation. No study has documented
the use of any of the other listed radiographic measurements to the outcome of treating
developmental coxa vara.
 Carroll K, Coleman S, Stevens PM: Coxa vara: Surgical outcomes of valgus osteotomies. J Pediatr Orthop 1997;17:220-
224.
 Cordes S, Dickens DR, Cole WG: Correction of coxa vara in childhood: The use of Pauwels' Y-shaped osteotomy. J Bone
Joint Surg Br 1991;73:3-6.

50. Figure 23 shows the radiograph of a 7 year-old girl


with a low thoracic-level myelomeningocele. She
has a history of skin ulcers over the apex of the
deformity, but her current skin condition is good.
Management of the spinal deformity should consist
of

1- physical therapy for hip stretching exercises.


2- kyphectomy and posterior fusion with
instrumentation.
3- anterior release and fusion using a rib strut
graft.
4- anterior release and strut grafting and posterior
fusion with instrumentation.
5- bracing.

PREFERRED RESPONSE: 2

DISCUSSION: This form of severe kyphosis results in intractable difficulties with sitting
position, compression of internal organs, and chronic skin breakdown. Kyphectomy and
posterior fusion with instrumentation, while associated with a high rate of complications,
provides one of the best solutions to this clinical dilemma. The other choices are either
completely ineffective or inadequate in managing this degree of deformity.
 Lindseth RE: Spine deformity in myelomeningocele. Instr Course Lect 1991;40:273-279.
 Sharrard J, Drennan JC: Osteotomy excision of the spine for lumbar kyphosis in older children with myelomeningocele. J
Bone Joint Surg Br 1972;54:50-60.
American Academy of Orthopaedic Surgeons
2004 pediatric orthopedic Surgery

51. A 10-year-old boy with an L1 myelomeningocele has a low-grade fever and a


swollen thigh that is warm to touch and erythematous. AP and lateral radiographs
are shown in Figures 24a and 24b. Management should
consist of

1- aspiration of the femoral


metaphysis.
2- IV antibiotics.
3- a biopsy.
4- immobilization in a well-
padded splint for 2 to 3 weeks.
5- immobilization in a spica cast
for 6 weeks.

PREFERRED RESPONSE: 4

DISCUSSION: Fractures of the long bones are common in patients with myelodysplasia, and the
frequency of fracture increases with higher level defects. Fractures also occur following surgery
and immobilization secondary to disuse osteoporosis. The response to the fracture (swelling,
fever, warmth, erythema) is often confused with infection, osteomyelitis, or cellulitis.
Management should consist of a short period of immobilization in a well-padded splint. Long-
term casting results in further osteopenia and repeated fractures.
 Lock TR, Aronson DD: Fractures in patients who have myelomeningocele. J Bone Joint Surg Am 1989;71:1153-1157.
 Kumar SJ, Cowell HR, Townsend P: Physeal, metaphyseal, and diaphyseal injuries of the lower extremities in children with
myelomeningocele. J Pediatr Orthop 1984;4:25-27.

52. A 6-year-old African-American boy with sickle cell


disease has had pain and limited use of his right
arm for the past 3 days. History reveals that he
sustained a humeral fracture approximately 3 years
ago. A lateral radiograph is shown in Figure 25.
Based on these findings, a presumptive diagnosis of
chronic osteomyelitis is made. What are the two
most likely organisms?

1- Haemophilus influenzae and Staphylococcus


aureus
2- Kingella kingae and Streptococcus pneumoniae
3- Staphylococcus aureus and Salmonella
4- Streptococcus pneumoniae and
Staphylococcus aureus
5- Salmonella and Shigella
American Academy of Orthopaedic Surgeons
2004 pediatric orthopedic Surgery

PREFERRED RESPONSE: 3

DISCUSSION: The risk of Salmonella osteomyelitis is much greater in patients with sickle cell
disease than the general population. The exact reason for this increased risk is still unclear, but it
appears to be associated with an increased incidence of gastrointestinal microinfarcts and
abscesses. Both Staphylococcus aureus and Salmonella have been mentioned as the most
prevalent causative organisms.
 Piehl FC, David RJ, Prugh SI: Osteomyelitis in sickle cell disease. J Pediatr Orthop 1993;13:225-227.
 Givner LB, Luddy RE, Schwartz AD: Etiology of osteomyelitis in patients with major sickle hemoglobinopathies. J Pediatr
1981;99:411-413.
 Epps CH Jr, Bryant DD III, Coles MJ, Castro O: Osteomyelitis in patients who have sickle-cell disease: Diagnosis and
management. J Bone Joint Surg Am 1991;73:1281-1294.
 Lovell and Winter's Pediatric Orthopaedics, ed 4, 1996.

53. A 7-year-old child is unresponsive, tachycardic, and has a systolic blood pressure of
50 mm Hg after being struck by a car. The patient is intubated and venous access is
obtained. The secondary survey reveals an unstable pelvis. Despite adequate
resuscitation, the patient continues to be hemodynamically unstable. What is the
best course of action?

1- CT of the pelvis to delineate the fracture pattern


2- Application of a pelvic sling
3- Radiographs of the long bones
4- Angiography to stem the flow of retroperitoneal bleeding
5- Immediate open reduction of the fracture

PREFERRED RESPONSE: 2

DISCUSSION: The patient is hemodynamically unstable, so any treatment should be aimed at


stabilization. Airway, breathing, and circulation are the most important areas to control initially;
the patient has been intubated and has adequate venous access. Despite fluid resuscitation, the
child remains hypotensive, indicating continued blood loss. With an unstable pelvic fracture
there can be significant hemorrhage. Decreasing the pelvic volume can decrease blood loss
related to the pelvic fracture. This can be done in the emergency department by applying a
pelvic sling. Other means of decreasing pelvic volume include a pelvic clamp, a simple anterior
frame pelvic external fixator, or a simple sheet tied around the pelvis. These maneuvers may
stabilize the patient so that further evaluation and treatment can be undertaken. All of the other
choices will delay stabilization and should be postponed until the patient is stabilized.
 Torode I, Zieg D: Pelvic fractures in children. J Pediatr Orthop 1985;5:76-84.
 Eichelberger MR, Randolph JG: Pediatric trauma-initial resuscitation, in Moore EE, Eisman B, Van Way CE (eds): Critical
Decisions in Trauma. St Louis, MO, CV Mosby, 1984, p 344.
 Ganz R, Krushell RJ, Jacob RP, Kuffer J: The antishock pelvic clamp. Clin Orthop 1991;267:71-78.
American Academy of Orthopaedic Surgeons
2004 pediatric orthopedic Surgery

54. A 3-year-old boy with severe cerebral palsy is unable to sit independently and does
not crawl. Examination reveals a 40-degree hip flexion contracture by the Thomas
test and 25 degrees of passive abduction. A radiograph of the pelvis shows
subluxation of both hips, with a migration index of 30%. Management should
consist of

1- application of a Pavlik harness.


2- botulinum toxin A injections to the adductor and iliopsoas muscles.
3- bilateral release of the adductor and iliopsoas muscles.
4- bilateral soft-tissue release and proximal femoral varus rotational osteotomies.
5- selective dorsal rhizotomy.

PREFERRED RESPONSE: 3

DISCUSSION: Progressive hip subluxation occurs in up to 50% of children with spastic


quadriparesis. The subluxation is the result of chronic muscle hypertonicity, especially in the
adductor muscle group. In time, the constant muscle tension will lead to dislocation, dysplastic
changes in the acetabulum, and erosive changes in the cartilage of the femoral head. Many of
these children will experience pain. Two recent studies have shown that early soft-tissue
releases can successfully prevent progressive subluxation in children who are younger than age 4
years and who have a Reimers index (migration index) of less than 40%. Botulinum toxin A
injections may reduce tone in the adductors for 4 to 6 months, but it is difficult to inject into the
iliopsoas. Additionally, there are no long-term studies documenting the efficacy of botulinum
toxin A to treat progressive hip subluxation in patients who have spastic quadriparesis. In
general, proximal femoral osteotomy, combined with soft-tissue release as necessary, is indicated
in older children (older than age 4 years) with progressive subluxation. Although selective
dorsal rhizotomy has been used in nonambulatory patients, outcomes are less well documented
than in ambulatory patients. There are no studies documenting the effect of selective dorsal
rhizotomy on progressive hip subluxation in nonambulatory children.
 Miller F, Cardoso Dias R, Dabney KW, et al: Soft-tissue release for spastic hip subluxation in cerebral palsy. J Pediatr
Orthop 1997;17:571-584.
 Cornell MS, Hatrick NC, Boyd R, et al: The hip in children with cerebral palsy: Predicting the outcome of soft tissue
surgery. Clin Orthop 1997;340:165-171.

55. The parents of a 3-year-old girl who has had pain and swelling in the right ankle for the
past 3 months now report that she has a limp and that the right knee and both ankles are
painful and swollen. The limp and difficulty walking are most severe in the morning
when the child first gets out of bed and are also more severe after extended walking. The
parents deny fever, chills, weight loss, or night pain. Examination shows mild swelling
and slightly restricted motion of the right knee and both ankles but is otherwise normal.
In addition to initiation of treatment, the child should be referred to which of the
following specialists?

1- Neurosurgeon
2- Ophthalmologist
American Academy of Orthopaedic Surgeons
2004 pediatric orthopedic Surgery

3- Oncologist
4- Otolaryngologist
5- Urologist

PREFERRED RESPONSE: 2

DISCUSSION: Pauciarticular juvenile rheumatoid arthritis (JRA) is the most common subgroup
of JRA. It typically presents between the ages of 2 to 4 years with a mild swelling of one to four
joints. Girls are affected four times more often than boys. The ankle and knee are commonly
involved, and limping is typically worse in the morning and after extended activity. The
diagnosis of pauciarticular JRA is typically one of exclusion because laboratory studies,
including erythrocyte sedimentation rate and rheumatoid factor, are usually within normal limits.
Pauciarticular JRA has the highest incidence of chronic uveitis, and in the subgroup with
elevated antinuclear antibody (ANA) titers, the incidence is 75%. In JRA, uveitis usually occurs
after the onset of synovitis but may precede the joint symptoms. At the early stage of uveitis, the
patient is asymptomatic. If the eye condition is not detected and treated, progressive loss of
vision may occur. Orthopaedic surgeons may be instrumental in making the diagnosis of
pauciarticular JRA. Pauciarticular JRA is not associated with conditions that require input from
the other specialists.
 Carey TP: Inflammatory arthritides: Juvenile rheumatoid arthritis, seronegative spondyloarthropathies, transient synovitis,
hemophilic arthropathy, in Fitzgerald RH, Kaufer H, Malkani AL (eds): Orthopaedics. St Louis, MO, Mosby, 2002, pp
1315-1321.
 Wright D: Juvenile idiopathic arthritis, in Morrissy RT, Weinstein SL (eds): Lovell and Winter's Pediatric Orthopaedics, ed
5. Philadelphia, PA, Lippincott Williams and Wilkins, 2001, pp 427-458.

56. A 9-year-old boy has pain over the midfoot medially with activity. Based on the
findings shown in Figures 26a and 26b, which of the following is considered the most
effective short-term management?

1- Crutches and no weight bearing


2- Core decompression
3- Shoe inserts
4- Temporary cast immobilization
5- Nonsteroidal anti-inflammatory drugs

PREFERRED RESPONSE: 4
American Academy of Orthopaedic Surgeons
2004 pediatric orthopedic Surgery

DISCUSSION: While Kohler's disease has a benign course, temporary cast immobilization has
been shown to result in a shortened duration of symptoms. Core decompression or other surgery
is not warranted because of the benign nature. Shoe inserts may be beneficial, but there are no
studies to support their use.
 Borges JL, Guile JT, Bowen JR: Kohler's bone disease of the tarsal navicular. J Pediatr Orthop 1995;15:596-598.
 Williams GA, Cowell HR: Kohler's disease of the tarsal navicular. Clin Orthop 1981;158:53-58.

57. During the first 2 years of life, which of the following actions is most responsible for
increasing structural stability of the physis?

1- The change from a flat to an undulating physis


2- The growth of the zone of Ranvier
3- Increased strength of the points of insertion of muscles onto bone
4- Increased penetration of proprioceptive nerve endings about the physis
5- Increased capillary penetration about the physis

PREFERRED RESPONSE: 2

DISCUSSION: The zone of Ranvier provides the earliest increase in strength of the physis.
During the first year of life, the zone spreads over the adjacent metaphysis to form a fibrous
circumferential ring bridging from the epiphysis to the diaphysis. This ring increases the
mechanical strength of the physis. The zone also helps the physis grow latitudinally. In turn, the
increased width of the physis helps the physis further resist mechanical forces. The change in
shape of the physis to its progressively more undulating form is also a factor in increasing
physeal strength, but this occurs over a longer period of time, as the child's activity level
increases. The undulations of the physis seen in some growth plates also add to stability but to a
lesser extent. The other changes contribute little toward increasing physeal strength.
 Burkus J, Ogden J: Development of the distal femoral epiphysis: A microscopic morphological investigation of the zone of
Ranvier. J Pediatr Orthop 1984;4:661-668.
 Shapiro F, Holtrop ME, Glimcher MJ: Organization and cellular biology of the perichondrial ossification groove of Ranvier:
A morphological study in rabbits. J Bone Joint Surg Am 1977;59:703-723.

58. Because the patient shown in Figure 27 can no


longer fit in shoes, treatment of the deformity
should consist of

1- physeal arrest.
2- soft-tissue debulking.
3- ray resection.
4- Chopart amputation.
5- distal phalanx amputation.

PREFERRED RESPONSE: 3
American Academy of Orthopaedic Surgeons
2004 pediatric orthopedic Surgery

DISCUSSION: In local gigantism, a ray resection allows proper fitting of shoes. The ray
resection narrows the foot and shortens the length. The foot may require further surgery with
growth. Debulking, physeal arrest, and distal phalanx amputation are unlikely to be effective.
 Turra S, Santini S, Cagnoni G, Jacopetti T: Gigantism of the foot: Our experience in seven cases. J Pediatr Orthop
1998;18:337-345.
 Guidera KJ, Brinker MR, Kousseff BG, et al: Overgrowth management in Klippel-Trenaunay-Weber and Proteus
syndromes. J Pediatr Orthop 1993;13:459-466.

59. Examination of a 9-year-old girl who injured her left


elbow in a fall reveals tenderness and swelling
localized to the medial aspect of the elbow. Motor
and sensory examinations of the hand are normal,
and circulation is intact. A radiograph is seen in
Figure 28. Management should consist of

1- Long arm cast immobilization.


2- Open reduction and internal fixation, followed
by cast immobilization.
3- Closed reduction and percutaneous pin fixation.
4- Anterior transposition of the ulnar nerve.
5- Excision of the loose fragment and repair of the
common flexor origin.

PREFERRED RESPONSE: 1

DISCUSSION: Avulsion fractures of the medial epicondyle are caused by a valgus stress applied
to the immature elbow and usually occur in children between the ages of 9 and 14 years. Long-
term studies have shown that isolated fractures of the medial epicondyle with between 5 to 15
mm of displacement heal well. Brief immobilization (1 to 2 weeks) in a long arm cast or splint
yields results similar to open reduction and internal fixation. Fibrous union of the fragment is
not associated with significant symptoms or diminished function. Surgical excision of the
fragment yielded the worst results in one study and should be avoided. Open reduction is best
reserved for those injuries in which the medial epicondylar fragment becomes entrapped in the
elbow joint during reduction and cannot be extracted by closed manipulation.
 Farsetti P, Potenza V, Caterini R, Ippolito E: Long-term results of treatment of fractures of the medial humeral epicondyle in
children. J Bone Joint Surg Am 2001;83:1299-1305.
 Josefsson PO, Danielsson LG: Epicondylar elbow fracture in children: 35-year follow-up of 56 unreduced cases. Acta
Orthop Scand 1986;57:313-315.

60. A 2-year-old child has refused to bear weight on his leg for the past 2 days. His parents
report that he will crawl, has no fever, and has painless full range of motion of his hip and
knee. Examination reveals no deformity or bruising, but there is mild swelling and
tenderness over the anterior tibia. C-reactive protein, WBC count, and erythrocyte
American Academy of Orthopaedic Surgeons
2004 pediatric orthopedic Surgery

sedimentation rate studies are normal. Radiographs are negative. What is the best course
of action?

1- Application of a long leg cast


2- Aspiration of the tibial metaphysis
3- Bone scan
4- MRI
5- Observation

PREFERRED RESPONSE: 1

DISCUSSION: Despite the negative radiographic findings, the child's age and presentation are
most consistent with a toddler's fracture. There is often not a witnessed injury. The differential
diagnosis of infection is unlikely given that the child is afebrile and shows no signs of illness.
Immobilization will make the child more comfortable and will often allow weight bearing.
Repeat radiographs at the end of treatment will show a healing fracture and confirm the
diagnosis. Aspiration of the tibial metaphysis would be indicated to obtain material for culture.
The bone scan and MRI would show abnormalities, but these studies are nonspecific, costly, and
time-consuming. Occasionally, oblique radiographs will show the fracture.
 Halsey MF, Finzel KC, Carrion WV, Haralabatos SS, et al: Toddler's fracture: Presumptive diagnosis and treatment. J
Pediatr Orthop 2001;21:152-156.
 Oudjihane K, Newman B, Oh KS, et al: Occult fractures in preschool children. Trauma 1988;28:858-860.

61. A 7-year-old girl sustains the fracture


shown in Figure 29a. Casting results in
uneventful healing. Ten months later, the
patient has a progressive valgus deformity
of the right lower extremity. A radiograph
is shown in Figure 29b. Management
should now consist of

1- observation.
2- proximal tibial osteotomy.
3- proximal tibial hemiepiphyseodesis.
4- a long leg brace with a varus-
producing strap.
5- MRI to map the extent of the osseous
physeal bridge.

PREFERRED RESPONSE: 1

DISCUSSION: Although fractures of the proximal tibial metaphysis in young children appear
innocuous, development of a progressive valgus deformity is possible despite adequate and
appropriate treatment. When treating a child with this injury, it is prudent to warn the parents
that a valgus deformity of the tibia may develop. The most likely cause is asymmetric growth of
American Academy of Orthopaedic Surgeons
2004 pediatric orthopedic Surgery

the proximal tibial physis. Because spontaneous angular improvement can be expected in most
patients, surgery to correct these deformities should be delayed at least 2 to 3 years and should be
limited to patients who have symptoms. There are no studies that document the efficacy of
bracing for this deformity.
 Tuten HR, Keeler KA, Gabos PG, et al: Posttraumatic tibia valga in children: A long-term follow-up note. J Bone Joint
Surg Am 1999;81:799-810.
 McCarthy JJ, Kim DH, Eilert RE: Posttraumatic genu valgum: Operative versus nonoperative treatment. J Pediatr Orthop
1998;18:518-521.

62. An obese 10-year-old boy has had left groin pain and a limp for the past 2 months.
Examination reveals decreased abduction and internal rotation. Laboratory studies
show normal renal function and an elevated thyroid-stimulating hormone (TSH)
level. AP and frog lateral radiographs of the pelvis are shown in Figures 30a and
30b. What is the best course of action?

1- Traction followed by reduction and pinning


2- In situ pinning of the left hip
3- In situ pinning of both hips
4- No weight bearing on the left side and nonsteroidal anti-inflammatory drugs
5- Femoral realignment osteotomy

PREFERRED RESPONSE: 3

DISCUSSION: The radiographs show a grade I slipped capital femoral epiphysis (SCFE) that is
classified as stable because the child is able to bear weight. The elevated TSH level indicates
possible hypothyroidism. SCFE usually occurs in boys age 12 to 14 years. Because of the
patient's young age and hypothyroidism, he is at increased risk for slippage of the contralateral
hip; therefore, prophylactic pinning of the uninvolved side also should be considered. Because
of the risk of slip progression, crutch treatment and nonsteroidal anti-inflammatory drugs are not
indicated. Realignment osteotomy is not indicated for grade I SCFE. Traction to reduce the slip,
followed by pinning, has been advocated for unstable slips but is not indicated here
 Loder RT, Wittenberg B, DeSilva G: Slipped capital femoral epiphysis associated with endocrine disorders. J Pediatr
Orthop 1995;15:349-356.
American Academy of Orthopaedic Surgeons
2004 pediatric orthopedic Surgery

 Wells D, King JD, Roe TF, Kaufman FR: Review of slipped capital femoral epiphysis associated with endocrine disease. J
Pediatr Orthop 1993;13:610-614.

63. A 7-year-old boy has had chronic left leg pain that is worse at night but is not
activity related. Use of nonsteroidal anti-inflammatory drugs for the past 6 months
has failed to provide relief. A CBC count with differential, erythrocyte
sedimentation rate, and C-reactive protein are within normal limits. Radiographs
and a CT scan are shown in Figures 31a through 31c. Management should consist
of

1- intralesional curettage followed by 6


weeks of parenteral antibiotics.
2- local curettage and autogenous bone graft.
3- neoadjuvant chemotherapy followed by
surgical resection and additional
chemotherapy
4- percutaneous radiofrequency coagulation.
5- closed needle biopsy.

PREFERRED RESPONSE: 4

DISCUSSION: Osteoid osteomas are painful bone lesions, with radiographs revealing a dense
sclerotic cortex surrounding a small radiolucency or nidus. Symptoms often are worse at night
but usually are not activity related. While treatment in the past has consisted of open en bloc
excision, current means of removal include percutaneous drilling under CT guidance and
percutaneous radiofrequency coagulation. Success rates of percutaneous treatment are
comparable to those seen following open procedures. The characteristic radiographic appearance
of this lesion usually obviates the need for biopsy. Because the lesion is not caused by pyogenic
organisms, antibiotics are not indicated.
 Donahue F, Ahmad A, Mnaymneh W, Pevsner NH: Osteoid osteoma: Computed tomography guided percutaneous excision.
Clin Orthop 1999;366:191-196.
 Rosenthal DI, Hornicek FJ, Wolfe MW, et al: Percutaneous radiofrequency coagulation of osteoid osteoma compared with
operative treatment. J Bone Joint Surg Am 1998;80:815-821.
American Academy of Orthopaedic Surgeons
2004 pediatric orthopedic Surgery

64. In patients with neurofibromatosis, what is the most important sign of impending
rapid progression of a spinal deformity?

1- Apical curve rotation


2- Anterior vertebral body erosions
3- Cervical spine involvement
4- Penciling of three or more ribs
5- Curve magnitude of greater than 50 degrees

PREFERRED RESPONSE: 4

DISCUSSION: Neurofibromatosis can progress very rapidly. Rib penciling is the only singular
prognostic factor. Significant progression has been observed in 87% of the curves with three or
more penciled ribs. The other factors are often present but do not have a high correlation with
rapid, severe progression.
 Crawford AH, Schorry EK: Neurofibromatosis in children: The role of the orthopaedist. J Am Acad Orthop Surg
1999;7:217-230.
 Durrani AA, Crawford AH, Chouhdry SN, Saifuddin A, Morley TR: Modulation of spinal deformities in patients with
neurofibromatosis type 1. Spine 2000;25:69-75.

65. The fracture shown in Figure 32 is strongly indicative of what diagnosis?

1- Osteogenesis imperfecta
2- Vitamin D-resistant rickets
3- Lead toxicity
4- Child abuse
5- Osteomyelitis

PREFERRED RESPONSE: 4

DISCUSSION: Fractures that occur through the primary spongiosa at the subphyseal region of
the metaphysis are highly specific for child abuse. On radiographic studies, the metaphyseal
lucency in these injuries may appear as either the so-called "bucket-handle" or "metaphyseal
corner" fracture. These fractures are not typical features of osteogenesis imperfecta or vitamin
D-resistant rickets. The ingestion of lead may lead to thick, transverse bands of increased
density at the distal metaphysis. Fractures in the subphyseal region of the metaphysis are not
typically seen in children who have osteomyelitis.
 Kocher MS, Kasser JR: Orthopaedic aspects of child abuse. J Am Acad Orthop Surg 2000;8:10-20.
 Kleinman PK, Marks SC, Blackbourne B: The metaphyseal lesion in abused infants: A radiologic-histopathologic study.
Am J Roentgenol 1986;146:895-905.
American Academy of Orthopaedic Surgeons
2004 pediatric orthopedic Surgery

66. Figures 33a and 33b show the radiographs of a 10-year-


old girl who reports a 4-month history of medial foot pain
after she was kicked while playing soccer. The pain is
worse with activity and partially relieved by rest.
Examination reveals tenderness directly over a prominent
navicular tuberosity. Management should consist of

1- calcaneal lengthening osteotomy with reefing of the


tibialis posterior tendon.
2- excision of the ossicle in the navicular and tibialis posterior tendon advancement.
3- talonavicular joint arthrodesis.
4- open reduction and internal fixation of the navicular.
5- activity and shoe modification and non-narcotic analgesics.

PREFERRED RESPONSE: 5

DISCUSSION: An accessory tarsal navicular is located at the medial tuberosity of the navicular
bone. Nearly all children and adolescents who have a symptomatic accessory tarsal navicular
bone become asymptomatic when they reach skeletal maturity. Initial management should
include activity restrictions, shoe modification to avoid pressure over the prominent navicular,
and non-narcotic analgesics. Although anecdotal, the use of arch supports may be helpful. When
pain is refractory to these methods, a short period of cast immobilization may be useful. Surgery
should be reserved for patients who have disabling symptoms despite a prolonged period of
nonsurgical management. When surgery is indicated, simple excision of the accessory navicular
is recommended.
 Sella EJ, Lawson JP, Ogden JA: The accessory navicular synchondrosis. Clin Orthop 1986;209:280-285.
 Bennett GL, Weiner DS, Leighley B: Surgical treatment of symptomatic accessory tarsal navicular. J Pediatr Orthop
1990;10:445-449.

67. An 18-month-old child with obstetrical palsy has a


maximum external rotation as shown in Figure 34.
The parents should be advised that without surgical
treatment the likelihood that glenoid dysplasia will
develop is approximately what percent?

1- 5%
American Academy of Orthopaedic Surgeons
2004 pediatric orthopedic Surgery

2- 10%
3- 20%
4- 30%
5- 70%

PREFERRED RESPONSE: 5

DISCUSSION: Based on the available literature, the probability of development of glenoid


dysplasia in the setting of a significant limitation of external rotation is close to 70%. Humeral
dysplasia is also likely and can be managed surgically. Efforts are being made to identify
procedures that will prevent glenoid dysplasia and help maintain function.
 Pearl ML, Edgerton BW: Glenoid deformity secondary to brachial plexus birth palsy. J Bone Joint Surg Am 1998;80:659-
667.
 Waters PM, Smith GR, Jaramillo D: Glenohumeral deformity secondary to brachial plexus birth palsy. J Bone Joint Surg
Am 1998;80:668-677.
 Zancolli EA, Zancolli ER Jr: Palliative surgical procedures in sequelae of obstetrical palsy. Hand Clin 1988;4:643-669.

68. A 10-year-old boy has a painful, swollen knee after falling off his bicycle.
Examination reveals no other injuries. Radiographs are shown in Figures 35a and
35b. Initial management of this fracture should consist of

1- open reduction and internal


fixation.
2- arthroscopic reduction and
internal fixation.
3- closed reduction following
evacuation of the
hemarthrosis and casting in
extension if reduction is
adequate.
4- closed reduction and casting
in 90 degrees of flexion.
5- excision of the fragment.

PREFERRED RESPONSE: 3

DISCUSSION: The radiographs show a minimally displaced fracture of the tibial eminence,
which is classified as a McKeever type II injury. In a number of studies, it has been found that
most of these fractures will reduce with extension of the knee. This is often made easier with
evacuation of the hemarthrosis. The position of knee immobilization is controversial, with some
authors preferring full extension and others preferring 20 degrees of flexion. Flexion to 90
degrees will further displace the fragment. If the fragment does not reduce or if the patient has a
McKeever type III or IV injury, reduction and internal fixation are required. This can be done
with either an open or an arthroscopic procedure. Excision of the fragment is not indicated.
 Meyers MH, McKeever FM: Fractures of the intercondylar eminence of the tibia. J Bone Joint Surg Am 1970;52:1677-
1684.
 Wiley JJ, Baxter MP: Tibial spine fractures in children. Clin Orthop 1990;255:54-60
American Academy of Orthopaedic Surgeons
2004 pediatric orthopedic Surgery

 Janarv PM, Westblad P, Johansson C, Hirsch G: Long-term follow-up of anterior tibial spine fractures in children. J Pediatr
Orthop 1995;15:63-68.
 Kuhn JE, Sailer MJ, Sterett WI, Hawkins RJ: Arthroscopic technique for the treatment of tibial spine fractures in the
skeletally immature patient. J Ortho Tech 1995;3:7-12.

69. Figures 36a and 36b show the radiographs of a 3-year old child who has a congenital
upper extremity deformity. Which of the following features would be a major
contraindication to a
centralization procedure?

1- Complete absence of the


thumb
2- Thrombocytopenia
3- Patient age of less than 5
years
4- Lack of elbow motion
5- Absence of the radial
artery

PREFERRED RESPONSE: 4

DISCUSSION: The patient has bilateral absent radii or radial clubhand. Patients who lack elbow
flexion take advantage of the hand position to reach their mouths, and a centralization procedure
would take away that ability. This procedure can be performed on patients with partial to
complete absence of the radius. A hypoplastic thumb can be addressed at a staged procedure; it
does not represent a contraindication to centralization. Complete thumb absence can be
addressed by pollicizing the index ray.
 Green DP: Operative Hand Surgery, ed 2. New York, NY, Churchill Livingstone, 1988, pp 269-271.
 Goldberg MJ, Meyn M: The radial clubhand. Orthop Clin North Am 1976;7:341-359.

70. Examination of a 4-year old child with obstetrical palsy reveals weak deltoids,
pectoralis major strength of 4-5, and normal hand function. External rotation of
the shoulder is limited. What is the most appropriate surgical procedure to restore
external rotation?

1- Distal rerouting of the biceps tendon


2- Glenohumeral fusion with external rotation
3- External rotation osteotomy of the proximal humerus
4- Latissimus dorsi and teres major transfer to the posterior rotator cuff
5- Latissimus dorsi and teres major transfer to the subscapularis
American Academy of Orthopaedic Surgeons
2004 pediatric orthopedic Surgery

PREFERRED RESPONSE: 4

DISCUSSION: Transfer of the latissimus dorsi and teres major to the posterior rotator cuff will
restore external rotation and some abduction. The procedure should be performed in children
who are approximately age 4 years, following spontaneous recovery and prior to significant
stiffness. External rotation osteotomy is more appropriate for an older child. Fusion should not
be performed until skeletal maturity. Distal biceps rerouting restores pronation for a supination
deformity. Latissimus dorsi and teres major transfer to the subscapularis would accentuate the
internal rotation. In younger patients without significant bony deformity, a subscapularis slide or
lengthening can restore external rotation.
 Strecker WB, McAllister JW, Manske PR, Schoenecker PL, Dailey LA: Sever-L'Episcopo transfers in obstetrical palsy: A
retrospective review of twenty cases. J Pediatr Orthop 1990;10:442-444.
 Hoffer MM, Wickenden R, Roper S: Brachial plexus birth palsies. J Bone Joint Surgery Am 1978;60:692-695.

71. A 7-month-old girl has had a severe flatfoot deformity since birth. The talar head is
prominent in the medial plantar arch of the foot. No other deformities of the spine
or extremities are present. Motor and sensory examinations of the extremities are
normal. Figures 37a through 37c show simulated weight-bearing AP and lateral
radiographs and a planter flexion lateral view. What is the most likely diagnosis?

1- Pes calcaneovalgus
2- Congenital vertical talus
3- Flexible pes planus
4- Peroneal spastic flatfoot
5- Congenital short Achilles tendon

PREFERRED RESPONSE: 2
DISCUSSION: Congenital vertical talus is a fixed dorsal dislocation of the talonavicular joint
with equinus of the ankle joint. The AP radiograph shows valgus of the midfoot and an
increased talocalcaneal angle; the lateral radiograph shows a vertically positioned talus and
equinus of the ankle joint, and the plantar flexion lateral view shows that the talonavicular joint
does not reduce. A line drawn through the long axis of the talus passes below the long axis of
the first metatarsal. Initial management should consist of serial casting to stretch the dorsal soft-
tissue structures; surgery eventually will be required to reduce the talonavicular joint. The
differential diagnosis of congenital vertical talus includes pes calcaneovalgus, flexible pes
planus, and peroneal spastic flatfoot. Pes calcaneovalgus, flexible pes planus, congenital short
Achilles tendon, and peroneal spastic flatfoot would not show resistent dorsal dislocation of the
navicular on the plantar flexion view.
 Kodros SA, Dias LS: Single-stage surgical correction of congenital vertical talus. J Pediatr Orthop 1999;19:42-48.
American Academy of Orthopaedic Surgeons
2004 pediatric orthopedic Surgery

 Stricker SJ, Rosen E: Early one-stage reconstruction of congenital vertical talus. Foot Ankle Int 1997;18:535-543.
72. A 12-year-old boy has severe left shoulder pain after being struck by an automobile.
A chest radiograph, AP and lateral radiographs, and a CT scan with three-
dimensional reconstruction of the scapula are shown in Figures 38a through 38d.
Management should consist of

1- closed reduction and a


shoulder spica cast.
2- open reduction and internal
fixation.
3- an abduction orthosis.
4- sling immobilization.
5- figure-of-8 splint.

PREFERRED RESPONSE: 4

DISCUSSION: Scapular body fractures in children are rare and are often associated with other
injuries of the chest and thorax. Management is generally nonsurgical, unless the injury is open,
and usually consists of support with a sling and gentle range-of-motion exercises to minimize
shoulder stiffness.
 Green N, Swiontkowski M: Skeletal Trauma in Children, ed 2. Philadelphia, PA, WB Saunders, 1998, vol 3, pp 319-341.
 Curtis RJ Jr, Rockwood CA Jr: Fractures and dislocations of the shoulder in children, in Rockwood CA Jr, Matsen FA III
(eds): The Shoulder. Philadelphia, PA, WB Saunders, 1990, vol 2, pp 991-1032.

73. Figure 39 shows the radiograph of a 4-month old infant who has been undergoing
weekly casting since birth for a congenital equinovarus deformity. Management
should now consist of
American Academy of Orthopaedic Surgeons
2004 pediatric orthopedic Surgery

1- Surgical correction.
2- Continued casting.
3- Botulinum toxin injection into
the gastrocnemius-soleus
complex.
4- An ankle-foot orthosis.
5- Observation with no further
treatment.

PREFERRED RESPONSE: 1

DISCUSSION: The radiograph shows the development of a rocker-bottom foot deformity. A


rocker-bottom foot occurs in the treatment of clubfoot when casting is continued in the presence
of a very tight gastrocnemius-soleus complex and an uncorrected hindfoot. While there are some
preliminary reports on using Botox injection and continued casting for the equinus deformity,
most authors recommend posterior or posterior medial release. Percutaneous tenotomy has been
recently recommended with the resurgence of the Ponsetti technique.
 Lehman WB, Atar D: Complications in the management of talipes equinovarus, in Drennan JC (ed): The Child's Foot and
Ankle. New York, NY, Raven Press, 1992, pp 135-136.
 Herring JA: Tachdjian's Pediatric Orthopedics, ed 4. Philadelphia, PA, WB Saunders, 2002, pp 927-935.
 Tachdjian MO: Pediatric Orthopedics, ed 2. Philadelphia, PA, WB Saunders, 1990, pp 2461-2564.

74. A 1-year-old infant has the hand deformities shown in Figure 40. What pathologic
process is the most likely cause of these deformities?

1- Genetic mutation
2- Teratogen exposure
3- Amniotic rupture
4- Iatrogenic influences
5- Developmental field disruption

PREFERRED RESPONSE: 3

DISCUSSION: Streeter's dysplasia is clearly related to rupture of the amnion in utero and is now
most commonly referred to as premature amnion rupture sequence. The deformities arise from
amniotic bands that wrap about protruding parts and from uterine packing because of the
accompanying oligohydramnios. Clubfoot can develop as a result of the latter mechanism.
Three limb involvement is most commonly seen, along with syndactyly. Treatment involves
resection of bands and Z-plasty of skin. The disease is not genetic and has not been related to
teratogen exposure or to iatrogenic influences such as amniocentesis. Developmental field
American Academy of Orthopaedic Surgeons
2004 pediatric orthopedic Surgery

disruption is not seen in this disease, and the growth potential of the involved parts is normal
unless neurovascular disruption has arisen from band formation.
 Foulkes GD, Reinker K: Congenital constriction band syndrome: A seventy-year experience. J Pediatr Orthop 1994;14:242-
248.

75. A 13-year-old girl with hallux valgus reports pain after playing basketball.
Radiographs show a hallux valgus angle of 20 degrees, an intermetatarsal angle of
11 degrees, a distal metatarsal articular angle of 10 degrees, and a congruent joint.
Management should consist of

1- shoe wear modification.


2- proximal crescentic osteotomy with distal soft-tissue realignment.
3- Mitchell osteotomy.
4- chevron osteotomy.
5- Keller procedure.

PREFERRED RESPONSE: 1

DISCUSSION: Shoe wear modification is the most appropriate management based on the
patient's age, high activity level, and relatively minor symptoms. She also has a mild hallux
valgus. Normal radiographic measurements are an intermetatarsal angle of less than 9 degrees, a
hallux valgus angle of less than 15 degrees, and a distal metatarsal articular angle of less than 9
degrees. Surgical procedures should be reserved for patients with more severe or progressive
deformities.
 Stephens HM: Bunions, in Fitzgerald RH, Kaufer H, Malkani AL (eds): Orthopaedics. St Louis, MO, Mosby, 2002, pp
1510-1519.
 Donley BG, Richardson GE: Disorders of the first ray, in Fitzgerald RH, Kaufer H, Malkani AL (eds): Orthopaedics. St
Louis, MO, Mosby, 2002, pp 1718-1731.

76. The mother of a 24-month-old girl reports that the child cannot rotate her right
forearm. She also notes delayed development, with the child first walking at 18
months. The child has a five-word vocabulary and has not begun using simple
phrases. Examination
reveals that the right
forearm is fixed in 80
degrees of pronation. The
remainder of the
examination of both upper
extremities is otherwise
normal. A radiograph is
shown in Figure 41. Which
of the following studies will
best aid in diagnosis?
American Academy of Orthopaedic Surgeons
2004 pediatric orthopedic Surgery

1- Renal ultrasound
2- Echocardiography
3- Chromosome analysis
4- Creatine phosphokinase
5- MRI of the forearm

PREFERRED RESPONSE: 3

DISCUSSION: The patient has classic radioulnar synostosis. Patients with this disorder
frequently have duplication of sex chromosomes. Synostosis is often seen in females with 48-
XXXX or 49-XXXXX in association with delayed development and mental retardation. In
males, it can be associated with 48-XXXY or 49-XXXXY. Radioulnar synostosis is not usually
associated with muscle disorders, congenital heart disease, or renal anomalies. MRI of the
forearm can reveal other soft-tissue anomalies, but this information is not particularly helpful in
planning therapy. Osteotomy is sometimes indicated to improve rotational position of the wrist,
but this patient's rotation is quite functional for everyday tasks, and rotational osteotomy is not
indicated.
 Rimoin DL, Connor JM, Pyeritz RE, Korf BR: Emery & Rimoin's Principles and Practice of Medical Genetics, ed 4. New
York, NY, Churchill Livingstone, 2002, pp 1196-1197.

77. Figure 42 shows the radiograph of a patient with spinal


muscular atrophy. Examination reveals good upper
extremity function, and she can tie her shoes and propel a
manual wheelchair. Posterior instrumentation and fusion
may result in

1- decreased longevity.
2- worsening of sitting balance.
3- worsening of pulmonary function.
4- temporary loss of upper extremity function.
5- poor patient or parent satisfaction.

PREFERRED RESPONSE:4

DISCUSSION: Spinal muscular atrophy is caused by an abnormal survival motor neuron gene
that prevents apoptosis of the motor nerves. Spinal fusion results in better sitting balance,
stabilized or improved pulmonary function, and high parental satisfaction, but it may result in at
least temporary loss of upper extremity function.
 Bentley G, Haddad F, Bull TM, Seingry D: The treatment of scoliosis in muscular dystrophy using modified Luque and
Harrington-Luque instrumentation. J Bone Joint Surg Br 2001;83:22-28.
 Furumasu J, Swank SM, Brown JC, Gilgoff I, Warath S, Zeller J: Functional activities in spinal muscular atrophy patients
after spinal fusion. Spine 1989;14:771-775.
 Granata C, Cervellati S, Ballestrazzi A, Corbascio M, Merlini L: Spine surgery in spinal muscular atrophy: Long-term
results. Neuromuscul Disord 1993;3:207-215.
American Academy of Orthopaedic Surgeons
2004 pediatric orthopedic Surgery

78. A 2-week-old infant has had diminished movement of the right upper extremity
since birth. Examination reveals weakness of shoulder abduction and external
rotation, elbow flexion, and forearm supination. Both pupils are equally round and
responsive to light. The remainder of the examination is normal. Radiographs of
the upper limb show a healing middle-third clavicle fracture. Management should
consist of

1- cervical spine radiographs.


2- MRI of the shoulder.
3- decompressive osteotomy of the clavicle.
4- microsurgical repair of the brachial plexus.
5- observation and range-of-motion exercises.

PREFERRED RESPONSE: 5

DISCUSSION: The patient has a classic Erb's palsy with weakness of the muscles innervated by
the fifth and sixth cervical roots. Horner syndrome, a poor prognostic indicator for recovery, is
absent in this infant. All infants with brachial plexus birth palsies initially should be monitored
for spontaneous recovery during the first 3 to 6 months of life. During this period of
observation, glenohumeral motion, especially external rotation, should be maintained. Many
infants will begin to show recovery within the first 6 to 8 weeks after birth and continue on to
normal function. The timing of microsurgery is controversial. A recent study found that the
outcome of microsurgical repair in patients who had no recovery of biceps function within 3
months after birth was similar compared to those who had recovery of biceps function between 3
and 6 months and no microsurgical repair. The author concluded that microsurgical repair was
effective in improving function in those infants who had no evidence of recovery of biceps
function within the first 6 months of life.
 Waters PM: Comparison of the natural history, the outcome of microsurgical repair, and the outcome of operative
reconstruction in brachial plexus birth palsy. J Bone Joint Surg Am 1999;81:649-659.
 Greenwald AG, Schute PC, Shiveley JL: Brachial plexus birth palsy: A 10-year report on the incidence and prognosis. J
Pediatr Orthop 1984;4:689-692.

79. Progressive paralysis is most likely to be seen in association with what type of
congenital vertebral abnormality?

1- Anterior failure of formation


2- Anterior failure of segmentation
3- Posterior failure of formation
4- Posterior failure of segmentation
5- Lateral failure of segmentation

PREFERRED RESPONSE: 1

DISCUSSION: Anterior failure of formation results in a progressive kyphosis that may lead to
cord compression and progressive neurologic deficit. Anterior failure of segmentation can also
American Academy of Orthopaedic Surgeons
2004 pediatric orthopedic Surgery

produce progressive kyphosis but usually is not severe enough to cause cord compression.
Posterior failure of formation is seen in conditions such as myelomeningocele in which the
neurologic deficit is generally stable. Lateral abnormalities and posterior failure of segmentation
are rarely associated with progressive neurologic deficit.
 McMaster MJ, Singh H: Natural history of congenital kyphosis and kyphoscoliosis: A study of one hundred and twelve
patients. J Bone Joint Surg Am 1999;81:1367-1383.
 Dubousset J: Congenital kyphosis and lordosis, in Weinstein SL (ed): The Pediatric Spine: Principles and Practice, ed 1.
New York, NY, Raven Press, 1994, pp 245-258.

80. A 9-year-old child has right groin pain after falling from a tree. Examination
reveals that the right leg is held in external rotation, and there is significant pain
with attempts at passive range of motion. Radiographs are shown in Figures 43a
and 43b. Management should consist of

1- Closed reduction and a double spica cast.


2- Skeletal traction for 3 weeks, followed by a double spica
cast.
3- Reduction and internal fixation crossing the physis if
necessary.
4- Reduction and internal fixation with primary bone
grafting.
5- Skin traction.
PREFERRED RESPONSE: 3
DISCUSSION: The complications of femoral neck fractures in children include osteonecrosis,
malunion, nonunion, and premature physeal closure. It is presumed that the risk of osteonecrosis
is directly related to the amount of displacement at the time of injury and is not affected by the
type of treatment. The risk of the other complications can be decreased depending on the type of
treatment. Anatomic reduction by either closed or open methods can reduce the risk of
malunion. The addition of internal fixation allows for maintenance of the reduction. In young
children who cannot comply with a partial or non-weight-bearing status, the addition of a spica
cast gives added protection.
 Canale ST: Fractures of the hip in children and adolescents. Orthop Clin North Am 1990;21:341-352.
 Hughes LO, Beaty JH: Fractures of the head and neck of the femur in children. J Bone Joint Surg Am 1994;76:283-292.
American Academy of Orthopaedic Surgeons
2004 pediatric orthopedic Surgery

81. An 8-year-old boy with moderate factor VIII hemophilia played kickball earlier in
the day and now reports progressively severe groin pain and is unable to walk.
Examination reveals marked paresthesias over the medial aspect of the distal tibia.
What is the most likely diagnosis?

1- Hemorrhage into the iliacus muscle


2- Hemorrhage into the quadriceps muscle
3- Severe hip joint hemarthrosis
4- Slipped capital femoral epiphysis
5- Avulsion fracture of the anterior superior iliac spine

PREFERRED RESPONSE: 1

DISCUSSION: The iliacus muscle is a frequent site of hemorrhage in patients with severe or
moderate hemophilia. In patients with moderate hemophilia, hemorrhage into the iliacus muscle
often follows play or sporting events that include forceful contraction of the hip flexor muscles.
An expanding iliacus hematoma compresses the adjacent femoral nerve, with one study reporting
60% complete femoral nerve palsy in hemophiliacs with an iliacus or iliopsoas hemorrhage.
Femoral nerve compression typically includes paresthesias in the distribution of the terminal
saphenous nerve branch. Hip joint hemarthrosis may occur, but this condition is not as frequent
in hemophiliacs as muscle hemorrhage into the iliacus muscle. More importantly, a hip joint
hemarthrosis is not associated with significant compression of the femoral nerve. Avulsion
fractures of the anterior superior iliac spine typically occur during adolescence and are not
associated with saphenous nerve paresthesias. Slipped capital femoral epiphysis does not have
an increased association with hemophilia and usually occurs during the adolescent years.
 Greene WB: Diseases related to the hematopoietic system, in Morrissy RT, Weinstein SL (eds): Lovell and Winter's
Pediatric Orthopaedics, ed 5. Philadelphia, PA, Lippincott Williams and Wilkins, 2001, pp 379-426.
 Gilbert MS, Radomisli TE: Therapeutic options in the management of hemophilic synovitis. Clin Orthop 1997;343:88-92.

82. The patient shown in Figure 44 reports that her toes hurt when she walks.
Management should consist of

1- Shoe modifications for comfort.


2- Fourth metatarsal lengthening.
3- Fourth metatarsophalangeal
fusion.
4- Fourth ray resection.
5- Soft-tissue releases and muscle
balancing.

PREFERRED RESPONSE: 1
American Academy of Orthopaedic Surgeons
2004 pediatric orthopedic Surgery

DISCUSSION: Brachymetatarsia is a congenital hypoplasia of one or more metatarsals.


Shortening of the fourth metatarsal is the most common form of brachymetatarsia and is often
bilateral. Taping and manipulative reduction attempts are ineffective, and extensor tenotomy and
capsulotomy are not likely to sufficiently correct the deformity. Fusion or metatarsal
lengthening result in complications and generally are not indicated. Shoe modifications, such as
extra-depth or extra-wide shoes, generally will improve symptoms. If pressure and trauma
persist in the older child, metatarsal lengthening or amputation may be indicated.
 Stevens PM: Toe deformities, in Drennan JC (ed): The Child's Foot and Ankle. New York, NY, Raven Press, 1992, p 195.
 Tachdjian MO: Pediatric Orthopedics, ed 2. Philadelphia, PA, WB Saunders, 1990, pp 2633-2637.

83. Figures 45a and 45b show the AP and lateral radiographs of a 15-year old patient
who is undergoing surgery to add 3 cm of length to the femur. Based on the
radiographic findings, what is the next most appropriate step in management?

1- The distraction rate should be maintained.


2- The distraction rate should be decreased.
3- The distraction should be reversed.
4- The distraction gap should be managed with bone grafting in an open procedure.
5- A repeat corticotomy should be performed.

PREFERRED RESPONSE: 2

DISCUSSION: Because the radiographs reveal poor regenerate bone, especially anteriorly and
laterally, the first step in management is to slow the distraction rate. If this does not solve the
problem, temporary reversal of the distraction, or "accordionization," can be used to induce a
greater healing response. Maintaining the same distraction rate will further impair regenerate
formation and delay healing. Bone grafting should be reserved as an option if decreasing the
distraction rate or alternating a week of compression with a week of distraction fails to improve
the callus formation. Repeat corticotomy is performed in patients with premature consolidation.
American Academy of Orthopaedic Surgeons
2004 pediatric orthopedic Surgery

 Raney EM: Limb-length discrepancy, in Fitzgerald RH, Kaufer H, Malkani AL (eds): Orthopaedics. St Louis, MO, Mosby,
2002, pp 1519-1526.
 Ilizarov GA: Pseudarthroses and defects of long tubular bones, in Transosseous Osteosynthesis: Theoretical and Clinical
Aspects of the Regeneration and Growth of Tissue. Berlin, Springer-Verlag, 1992, p 453.

84. Examination of a 9-year-old boy reveals a right thoracic prominence on forward


flexion. Neurologic examination is normal, and no other abnormalities are noted.
AP radiographs reveal a 30-degree right thoracic curve. Initial management should
consist of

1- anteroposterior fusion.
2- observation.
3- MRI of the spine.
4- an orthosis.
5- instrumentation without fusion.

PREFERRED RESPONSE: 3

DISCUSSION:The patient has juvenile scoliosis. MRI has shown an association between
juvenile scoliosis and intraspinal abnormalities, most often syringomyelia and Arnold-Chiari
malformations. All juvenile curves greater than 20 degrees should be evaluated with MRI
despite the absence of neurologic findings.
 Weinstein SL (ed): The Pediatric Spine: Principles and Practice, ed 1. New York, NY, Raven Press, 1994, pp 685-705
 Nohria V, Oakes WJ: Chiari I malformation: A review of 43 patients. Pediatr Neurosurg 1990-91;16:222-227.
 Goldberg CJ, Moore DP, Fogarty EE, Dowling FE: Left thoracic curve patterns and their association with disease. Spine
1999;24:1228-1233.

85. A 13-year-old boy is comatose and has irregular breathing after being struck by a
car while riding his bicycle. Auscultation suggests a pneumothorax on the right side
and swelling about the right arm and leg. Initial management should consist of

1- careful neurologic evaluation because of suspected brain injury.


2- CT of the brain because of suspected subdural hematoma.
3- insertion of an internal jugular vein central line for vascular access.
4- airway control, placement of vascular access lines, and cervical spine
radiographs.
5- a chest tube and chest radiograph.

PREFERRED RESPONSE: 4

DISCUSSION: The first priority is to gain control of the airway with intubation. Following
intubation, management should consist of ventilation and placement of a chest tube if needed,
vascular access and circulatory stabilization, radiographs of the cervical spine and chest, and CT
of the brain.
American Academy of Orthopaedic Surgeons
2004 pediatric orthopedic Surgery

 American College of Surgeons Committee on Trauma. Advanced Trauma Life Support Course. Instructor's Manual.
Chicago, IL, American College of Surgeons, 1984.
 Eichelberger MR, Randolph JG: Pediatric trauma: An algorithm for diagnosis and therapy. J Trauma 1983;23:91-97.

86. Overgrowth of a limb in a patient with neurofibromatosis type 1 (NF1) is most likely
associated with the presence of

1- dural ectasia.
2- Lisch nodules.
3- juvenile xanthogranuloma.
4- malignant peripheral nerve sheath tumor.
5- plexiform neurofibroma.

PREFERRED RESPONSE: 5

DISCUSSION: Plexiform neurofibromas are lesions found in patients with NF1. Clinical reports
show the prevalence of plexiform neurofibroma to be 20% to 30% but increases to 40% when
imaging studies are routinely obtained. The lesions are characterized by diffuse hypertrophy of
the involved nerves but with preservation of the nerves' fascicular organization. The lesions may
involve the dermis or may arise in the deeper structures. Palpation of a dermal lesion provokes
an image of a "bag of worms." Plexiform neurofibromas may cause disfigurement and
hyperpigmentation of the overlying skin. The lesions also can cause diffuse hypertrophy of the
soft tissue and bone, with resultant changes ranging from a relatively minor limb-length
discrepancy to gigantism of the entire extremity. Dural ectasia is frequently found in patients
with NF1. Therefore, MRI should be obtained prior to planning spinal procedures in these
patients; however, dural ectasia is not the cause of limb overgrowth. Lisch nodules are benign
hamartomas of the iris. The lesions are uncommon during early childhood but are found in all
adults with NF1. Juvenile xanthogranuloma has a low occurrence rate in patients with NF1; its
presence is associated with juvenile chronic myeloid leukemia. Malignant peripheral nerve
sheath tumors, formally called neurofibrosarcoma, result from malignant degeneration of a
plexiform neurofibroma. This condition occurs in up to 4% of patients with NF1. Localized
pain, an enlarging mass, or progressive neurologic symptoms suggest a malignant peripheral
nerve sheath tumor in a patient with NF1. However, progressive neurologic symptoms also may
occur with benign growth of a plexiform neurofibroma.
 Alman BA, Goldberg MJ: Syndromes of orthopaedic importance, in Morrissy RT, Weinstein SL (eds): Lovell and Winter's
Pediatric Orthopaedics, ed 5. Philadelphia, PA, Lippincott Williams and Wilkins, 2001, pp 287-338.
 Greene WB: Neurofibromatosis type I, in Fitzgerald RH, Kaufer H, Malkani AL (eds): Orthopaedics. St Louis, MO,
Mosby, 2002, pp 1584-1588.

87. A 14-year-old girl with a right thoracic curve from T4 through L2 measuring 78
degrees is scheduled to undergo posterior spinal fusion for scoliosis. The surgical
plan is to fuse from T3 through L2, using pedicle screws at L2 and about the apex at
T8. What neural monitoring modality is most likely to identify a reversible
neurologic deficit during surgery?

1- Electromyography following stimulation of the lumbar pedicle screws


American Academy of Orthopaedic Surgeons
2004 pediatric orthopedic Surgery

2- Electromyography with stimulation of the thoracic pedicle screw


3- Motor-evoked potentials of the lower extremities
4- Somatosensory-evoked potentials of the upper extremities
5- Somatosensory-evoked potentials of the lower extremities

PREFERRED RESPONSE: 4

DISCUSSION: Neural monitoring during scoliosis surgery was initially developed to avoid the
devastating effects of spinal cord injury, particularly paraplegia. Monitoring in some form has
become standard for this type of surgery. Somatosensory-evoked potentials in the lower
extremities will detect many but not all neurologic difficulties with the spinal cord. Anterior
spinal cord vascular disruption also can be detected by monitoring motor potentials.
Electromyography following stimulation of lumbar pedicle screws can prevent nerve root injury
that is the result of misplacement of the screws. This is best documented in the lumbar spine and
has not been routinely used in the thoracic spine. The most common neural deficits following
spinal surgery, however, are in the upper extremities because of the positioning of the patient in
the prone position for long periods. In Schwartz and associates series of 500 patients, impending
upper extremity neural injury was detected by somatosensory-evoked potentials in 18 (3.6%)
patients. In contrast, lower extremity deficits were detected by combined motor- and sensory-
evoked potentials in only 2 (0.4%) out of 500 patients in Padberg and associates series. Neural
compression in the upper extremity can be easily detected by somatosensory-evoked potentials,
and injury can be prevented by repositioning the patient.
 Padberg AM, Wilson-Holden TJ, Lenke LG, Bridwell KH: Somatosensory- and motor-evoked potential monitoring without
wake-up test during idiopathic scoliosis surgery: An accepted standard of care. Spine 1998;23:1392-1400.
 Schwartz DM, Drummond DS, Hahn M, Ecker MI, Dormans JP: Prevention of positional brachial plexopathy during
surgical correction of scoliosis. J Spinal Disord 2000;13:178-182.

88. Figure 46 shows the radiograph of an obese 12-year-old boy who has had left hip
pain for the past 3 months. What is the best course of action?

1- Decreased activities and


physical therapy
2- Left hip reduction and
internal fixation
3- Left hip pinning in situ
4- Bilateral hip pinning in situ
5- Spica cast immobilization

PREFERRED RESPONSE: 3

DISCUSSION: The patient has an obvious slipped capital femoral epiphysis of the left hip for
which the recommended treatment is percutaneus pinning in situ. Development of a contralateral
American Academy of Orthopaedic Surgeons
2004 pediatric orthopedic Surgery

slip is less likely at this age; therefore, observation of the right hip is indicated because there is
no general agreement regarding prophylactic fixation. Typically, there is no role for spica
casting. Physical therapy is not indicated as a primary treatment, and reduction is
contraindicated, as it has been associated with osteonecrosis.
 Loder RT, Aronsson DD, Greenfield ML: The epidemiology of bilateral slipped capital femoral epiphysis: A study of
children in Michigan. J Bone Joint Surg Am 1993;75:1141-1147.
 Aronsson DD, Karol LA: Stable slipped capital femoral epiphysis: Evaluation and management. J Am Acad Orthop Surg
1996;4:173-181.
 Hurley JM, Betz RR, Loder RT, Davidson RS, Alburger PD, Steel HH: Slipped capital femoral epiphysis: The prevalence of
late contralateral slip. J Bone Joint Surg Am 1996;78:226-230.
 Loder RT, Aronson DD, Dobbs MB, Weinstein SL: Slipped capital femoral epiphysis. J Bone Joint Surg Am
2000;82:1170-1188.

89. A 7-year-old boy sustained a head contusion and small bowel injuries in a motor
vehicle accident in which he was wearing a lap belt. He subsequently required a
bowel resection. Six weeks after the accident, his parents note a painful mass in his
lower back. His neurologic examination is normal. A radiograph and CT scans are
shown in Figures 47a through 47c. Definitive management should now consist of

1- Transcutaneous electrical stimulation and a lumbar corset.


2- Transforaminal interbody fusion.
3- Posterior instrumented L2-L3 reduction and fusion.
4- Anterior interbody fusion with a cage.
5- Spine extension bracing.

PREFERRED RESPONSE: 3

DISCUSSION: The posttraumatic lumbar kyphotic deformity will not remodel and is likely to
worsen with time because the central line of gravity lies anterior to the deformity and the
ligamentous disruption will not heal. The worsening deformity also puts the patient at some risk
for future neurologic damage.
 Ebraheim NA, Savolain ER, Southworth SR, et al: Pediatric lumbar seat belt injuries. Orthopedics 1991;14:1010-1013.
 Taylor JA, Eggli KD: Lap belt inhuries of the lumbar spine in children: A pitfall in CT diagnosis. Am J Rad
1988;150:1355-1358.
American Academy of Orthopaedic Surgeons
2004 pediatric orthopedic Surgery

90. A 5-month-old girl with arthrogryposis has a limb-length discrepancy. Examination


and radiographs reveal unilateral hip dislocation. Management should consist of

1- A Pavlik harness.
2- Observation.
3- Closed reduction and a spica cast.
4- Open reduction and femoral shortening.
5- Open reduction.

PREFERRED RESPONSE: 5

DISCUSSION: In this age group of patients with arthrogryposis, open reduction through a
medial approach is generally recommended. Open reduction through an anterior approach is
reserved for patients in which a medial approach has failed or for older patients who require
simultaneous femoral shortening and/or pelvic osteotomy. Closed treatment of unilateral hip
dislocation in association with arthrogryposis is rarely successful. In bilateral hip dislocation
associated with arthrogrypsis, the consensus is that the hips are best left unreduced because of
the difficulty in obtaining excellent clinical and radiographic results bilaterally.
 Staheli LT, Chew DE, Elliot JS, Mosca VS: Management of hip dislocations in children with arthrogryposis. J Pediatr
Orthop 1987;7:681-685.
 Szoke G, Staheli LT, Jaffe K, Hall JG: Medial-approach open reduction of hip dislocation in amyoplasia-type
arthrogryposis. J Pediatr Orthop 1996;16:127-130.
 Richards BS (ed): Orthopaedic Knowledge Update: Pediatrics. Rosemont, IL, American Academy of Orthopaedic
Surgeons, 1996.

91. A 10-year-old boy who has had progressive low back and right buttock pain for the
past 3 days is now unable to bear weight on the right side secondary to pain. He has
a temperature of 101.3 degrees F (38.5 degrees C). Examination reveals full hip
range of motion; but he reports pain on the right side with external rotation. Pain is
elicited with compression of the iliac wings and with direct palpation of the right
sacroiliac (SI) joint. An MRI scan of the pelvis shows no abscess, but there is
inflammation of the SI joint. Management should consist of

1- Nonsteroidal anti-inflammatory drugs.


2- Bed rest.
3- Bed rest, blood cultures, and IV antibiotics.
4- Right SI joint aspiration.
5- Right SI joint fusion.

PREFERRED RESPONSE: 3

DISCUSSION: The clinical presentation and MRI findings are consistent with an acute infection
of the SI joint. Bed rest and nonsteroidal anti-inflammatory drugs alone are insufficient to treat
the problem. Staphylococcus aureus is the causative organism in most of these infections;
therefore, unless there is an unusual factor in the history such as IV drug use, immune system
compromise, or unusual travel, SI joint aspiration is unnecessary. It is often difficult to enter the
American Academy of Orthopaedic Surgeons
2004 pediatric orthopedic Surgery

SI joint, even under radiographic guidance. Management should consist of hospital admission
and IV antibiotics. Blood cultures may be positive and should be obtained prior to starting
antibiotics. Surgical fusion of the SI joint is not indicated.
 Morrissey RT: Bone and Joint Sepsis in Pediatric Orthopaedics. Philadelphia, PA, JB Lippincott, 1990. Beaupre A, Carroll
N: The three syndromes of iliac osteomyelitis in children. J Bone Joint Surg Am 1979;61:1087-1092.
 Reilly JP, Gross RH, Emans JB, Yngve DA: Disorders of the sacro-iliac joint in children. J Bone Joint Surg Am
1988;70:31-40.

92. Figures 48a and 48b show the elbow radiographs of a 5-year-old boy who fell from a
tree after dinner. Examination reveals that he is unable to extend his wrist.
Management should consist of immediate

1- Closed reduction and pinning of the


fracture in the operating room.
2- Closed reduction, pinning of the
fracture, and radial nerve exploration
in the operating room.
3- Closed reduction and splinting in the
emergency department.
4- Splinting of the arm and observation
of neurovascular function, followed
by closed reduction and pinning in the
operating room the following
morning.
5- Olecranon skeletal traction for
reduction, followed by casting when the reduction has been stabilized by callus.

PREFERRED RESPONSE: 4

DISCUSSION: In the absence of vascular compromise, there has been no proven value to
proceeding immediately to surgery, especially when the patient has a full stomach and runs a
significant risk of perioperative aspiration. It would be more prudent to wait until the next
morning with a surgical plan of closed reduction and pinning. Open reduction should be
reserved for the unusual case of where closed treatment has not been successful. The implication
that there may be a radial nerve injury associated with this fracture does not alter the treatment
plan, and with a high level of certainty would be expected to resolve. Attempting closed
reduction in the emergency department creates the opportunity for uncertain results and is not
tolerated well by most patients. Skeletal traction, with its associated lengthy hospitalization and
the technical difficulties associated with both the traction and radiographic evaluations, has
fallen into disfavor for typical clinical situations.
 Iyengar SR, Hoffinger SA, Townsend DR: Early versus delayed reduction and pinning of type III displaced supracondylar
fractures of the humerus in children: A comparative study. J Orthop Trauma 1999;13:51-55.
 Leet AI, Frisancho J, Ebramzadeh E: Delayed treatment of type 3 supracondylar humerus fractures in children. J Pediatr
Orthop 2002;22:203-207.
American Academy of Orthopaedic Surgeons
2004 pediatric orthopedic Surgery

93. In addition to the radiographic features seen in Figures 49a and 49b, this patient
will most likely have which of the following findings?

1- Microcephaly
2- 1,25 dihydroxy vitamin D deficiency
3- Normal dentition
4- Anemia
5- Developmental delays

PREFERRED RESPONSE: 4

DISCUSSION: The radiographs show the characteristic features of osteopetrosis. The condition
results from defective resorption of immature bone by osteoclasts. There are three distinct
clinical forms: (1) infantile-malignant, which is autosomal recessive and fatal in the first few
years of life if untreated; (2) intermediate autosomal recessive; and (3) autosomal dominant.
These conditions do not follow a malignant course, and patients have normal life expectancy
with orthopaedic problems and anemia. In the malignant form, the clinical features include
frequent fractures, macrocephaly, progressive deafness and blindness, hepatosplenomegaly, and
severe anemia beginning in early infancy or in utero. Deafness and blindness are generally
thought to represent effects of pressure on nerves and usually occur later in life. The anemia is
caused by encroachment of bone on marrow, resulting in obliteration, and the
hepatosplenomegaly is caused by compensatory extramedullary hematopoiesis. Dental caries
and abscesses, as well as osteomyelitis of the mandible, are also seen. Most patients have
normal intelligence. Treatment of the malignant form includes high dose 1,25 dihydroxy vitamin
D with a low-calcium diet to stimulate bone resorption, not because there are vitamin
deficiencies. Bone marrow transplant has also been successful.
 Herring JA: Tachdjian's Pediatric Orthopedics, ed 4. Philadelphia, PA, WB Saunders, 2002, p 1550. Zaleske DJ:
Metabolic and endocrine abnormalities, in Morrissy RT, Weinstein SL (eds): Lovell and Winter's Pediatric Orthopaedics, ed
5. Philadelphia, PA, Lippincott Williams and Wilkins, 2001, pp 212-214.
 Kaplan FS, August CS, Fallon MD, et al: Successful treatment of infantile malignant osteopetrosis by bone-marrow
transplantation: A case report. J Bone Joint Surg Am 1988;70:617-623.

94. A biopsy of the involved physis in a patient with slipped capital femoral epiphysis
(SCFE) would most likely reveal

1- A dense perichondral ring.


2- An abnormally thick zone of proliferation.
3- Increased undulations or irregularity of the growth plate.
4- Granulation tissue between the columns in the zone of hypertrophy.
5- Normal proteoglycan content.
American Academy of Orthopaedic Surgeons
2004 pediatric orthopedic Surgery

PREFERRED RESPONSE: 4

DISCUSSION: Vascular invasion, histologically similar to granulation tissue, has been noted
between the columns in the zone of hypertrophy, leading to the theory of microtrauma as an
etiology. SCFE is also associated with conditions that increase the height of the zone of
hypertrophy, including the adolescent growth spurt and endocrinopathies. The perichondral ring
has been shown to decrease in thickness with age. Normal undulations in the growth plate also
decrease during this time, possibly further destabilizing the physis. Abnormal accumulations of
proteoglycan have been reported.
 Chung SM, Batterman SC, Brighton CT: Shear strength of the human femoral capital epiphyseal plate. J Bone Joint Surg
Am 1976;58:94-103.
 Raney EM, Ogden JA: Slipped capital femoral epiphysis. Current Ortho 1995;9:111-116.

95. A 12-year-old girl has a 4-cm limb-length discrepancy following a fracture of the left
distal femur 2 years ago. Examination reveals 18 degrees of genu valgum on the
involved side, with 7 degrees of genu valgum on the opposite side. Radiographs
show that the left distal femoral growth plate is now closed; however, the tibial
growth plate is still open. Her bone age matches her chronologic age. Management
should consist of

1- Closed right femoral shortening.


2- Left femoral lengthening.
3- Left tibial lengthening.
4- A left 3-cm shoe lift.
5- Epiphyseodesis of the right distal femur and proximal tibia.

PREFERRED RESPONSE: 2

DISCUSSION: The patient has a projected limb-length discrepancy of 7 cm. This includes the 4
cm she already has, plus 3 cm expected growth of the uninvolved distal femur during the 3 years
of growth she has remaining. She also has moderate limb deformity. Femoral lengthening is
considered the treatment of choice because it can address both the limb-length discrepancy and
the deformity. Epiphyseodesis will not result in limb-length equality at maturity, with only
approximately 1.8 cm of equalization expected from this procedure. Use of closed femoral
shortening of 7 cm runs the risk of weakening the quadriceps on the normal side and will leave
the patient with a remaining residual valgus deformity. Tibial lengthening will leave the knees at
different levels. A shoe lift can be prescribed as a temporary measure but is not a good long-
term solution.
 Westh RN, Menelaus MB: A simple calculation for the timing of epiphyseal arrest: A further report. J Bone Joint Surg Br
1981;63:117-119.
 Sasso RC, Urquhart BA, Cain TE: Closed femoral shortening. J Pediatr Orthop 1993;13:51-56.
 Nordsletten L, Holm I, Steen H, Bjerkreim I: Muscle function after femoral shortening osteotomies at the subtrochanteric
and mid-diaphyseal level: A follow-up study. Arch Orthop Trauma Surg 1994;114:37-39.
American Academy of Orthopaedic Surgeons
2004 pediatric orthopedic Surgery

96. The parents of a previously healthy 3-year-old child report that she refused to walk
on awakening. Examination later in the day reveals that the patient can walk but
with a noticeable limp. She has a temperature of 99.5 degrees F (37.5 degrees C).
Range of motion measurements are shown in Figure 50. An AP pelvis radiograph is
normal. Laboratory studies show a WBC count of 9,000/mm3 and an erythrocyte
sedimentation rate of 10 mm/h. Management should consist of

1- Observation.
2- Technetium Tc 99m bone scan.
3- MRI of the pelvis.
4- Aspiration of the hip joint.
5- IV antibiotics.

PREFERRED RESPONSE: 1

DISCUSSION: The patient has the typical history and presentation of transient synovitis of the
hip, a condition that is more common in children age 2 to 5 years but which may affect children
up to 12 years. The discomfort typically is noted on awakening, and the child will refuse to
walk. Later in the day, the pain commonly improves and the child can walk but will have a limp.
Mild to moderate restriction of hip abduction is the most sensitive range-of-motion restriction.
The extent of the evaluation for transient synovitis depends on the intensity and duration of
symptoms. Because she has been afebrile for the past 24 hours, observation is the management
of choice. In the differential diagnosis of suspected transient synovitis, septic arthritis of the hip
is the primary disorder to exclude. Osteomyelitis of the proximal femur also should be
considered. In most patients, clinical examination will differentiate of these disorders to a
reasonable certainty. Plain radiographs are normal in the early stage of an infectious process.
Ultrasonography shows increased fluid in the hip joint in both transient synovitis and septic
arthritis. MRI can differentiate the two conditions; however, this test would require general
anesthesia and is not required in most patients in this age group. If a child with transient
synovitis has a concurrent infectious process such as an upper respiratory tract infection or otitis
media, the temperature will most likely be elevated. In this situation, a full evaluation for an
infectious process and initiation of IV antibiotics should be considered. This would include
radiographs, CBC count, erythrocyte sedimentation rate, blood cultures, aspiration of the hip
joint, and IV antibiotics.
 Del Beccaro MA, Champoux AN, Bockers T, Mendelman PM: Septic arthritis versus transient synovitis of the hip: The
value of screening laboratory tests. Annals Emerg Med 1992;21:1418-1422.
 Kehl DK: Developmental coxa vara, transient synovitis, and idiopathic chondrolysis of the hip, in Morrissy RT, Weinstein
SL (eds): Lovell and Winter's Pediatric Orthopaedics, ed 5. Philadelphia, PA, Lippincott Williams and Wilkins, 2001, pp
1035-1058.
American Academy of Orthopaedic Surgeons
2004 pediatric orthopedic Surgery

97. A 12-year-old boy with hemophilia A and no known inhibitors sustains a tibia
fracture and has pain with passive motion of the deep toe flexors. Appropriate
management should consist of

1- Emergency fasciotomy, followed by factor VIII replacement.


2- Cryoprecipitate, followed by assessment of compartment pressures.
3- Factor VIII replacement, followed by assessment of compartment pressures.
4- Physical therapy to prevent contractures.
5- Splinting, ice, and elevation.

PREFERRED RESPONSE: 3

DISCUSSION: In a patient with hemophilia, factor replacement followed by assessment of


compartment pressures is essential. If the patient has inhibitors, the problem is more difficult.
Porcine factor has been helpful in patients with inhibitory antibodies. Recent evidence points to
using activated factor VII and bypassing the intrinsic pathway. Desmopressin is an adjunct to
therapy but is not as effective as factor VII.
 Dumontier C, Sautet A, Man M, Bennani M, Apoil A: Entrapment and compartment syndromes of the upper limb in
haemophilia. J Hand Surg Br 1994;19:427-429.
 Carr ME Jr, Loughran TP, Cardea JA, Smith WK, Kuhn JG, Dottore MV: Successful use of recombinant factor VIIa for
hemostasis during total knee replacement in a severe hemophiliac with high-titer factor VIII inhibitor. Int J Hematol
2002;75:95-99.
 Chuansumrit A, Isarangkura P, Angchaisuksiri P, et al: Controlling acute bleeding episodes with recombinant factor VIIa in
haemophiliacs with inhibitor: Continuous infusion and bolus injection. Haemophilia 2000;6:61-65.

98. A 10-year-old boy with spastic diplegic cerebral palsy walks in a crouched position
with the hips and knees flexed. Maximum knee flexion is 15 degrees during early
swing phase. Instrumented gait analysis shows quadriceps activity from terminal
stance throughout swing phase. Treatment should consist of

1- Proximal release of the proximal tendon of the rectus femoris.


2- Release of the distal tendon of the rectus femoris.
3- Transfer of the distal rectus femoris tendon.
4- V-Y lengthening of the quadriceps tendon.
5- Z lengthening of the patellar tendon.

PREFERRED RESPONSE: 3

DISCUSSION: The rectus femoris muscle spans two joints and is active during running,
sprinting, and walking at a fast pace during the preswing and early swing phase of gait. In these
situations, the muscle helps to generate power to initiate hip flexion while absorbing or
controlling the rate of knee flexion during early swing phase. Quadriceps activity, including the
rectus femoris, is not normally needed when walking at a routine cadence. However, rectus
femoris activity is commonly noted during preswing and the swing phase in patients with
cerebral palsy, particularly those with diplegia. In an effort to initiate swing phase, the rectus
femoris is "overactive." As a result, the knee flexion that commonly occurs at terminal stance
American Academy of Orthopaedic Surgeons
2004 pediatric orthopedic Surgery

and initial swing is restricted. Instead of achieving the normal 50 to 60 degrees of flexion during
early swing, this patient's knee flexion is limited to 15 degrees. The goal of treatment is to retain
rectus femoris activity for initiation of hip flexion but to diminish its restraint on knee flexion.
Studies have shown that transfer of the distal rectus femoris tendon provides more flexion of the
knee during the swing phase of gait than simply releasing the tendon. V-Y lengthening of the
quadriceps tendon or a Z lengthening of the patellar tendon causes too much weakening of the
quadriceps muscle and worsens the crouch deformity. In addition to transfer of the rectus
femoris tendon, other procedures are often done concomitantly to obtain the best balance and
realignment of hip-knee-ankle activity.
 Aiona MD: Guidelines for managing lower extremity problems in cerebral palsy, in Fitzgerald RH, Kaufer H, Malkani AL
(eds): Orthopaedics. St Louis, MO, Mosby, 2002, pp 1534-1541.
 Chambers H, Laure A, Kaufman K, Cardelia M, Sutherland D: Prediction of outcome after rectus femoris surgery in
cerebral palsy: The role of cocontraction of the rectus femoris and vastus lateralis. J Pediatr Orthop 1998;18:703-711.
 Ounpuu S, Muik E, Davis RB, Gage JR, Deluca PA: Rectus femoris surgery in children with cerebral palsy. J Pediatr
Orthop 1993;13:331-335.

99. A 4-year-old boy with arthrogryposis has little active motion of his knees or elbows.
Both elbows are in full extension with good triceps strength, but he is unable to
bring his hand to his face or feed himself. Management should consist of

1- Observation.
2- Serial casting.
3- Flexion osteotomy of the distal humerus.
4- Posterior soft-tissue elbow release and anterior triceps transfer.
5- Adaptive equipment.

PREFERRED RESPONSE: 4

DISCUSSION: Elbow release and triceps transfer to restore motion can be performed in children
who are age 4 years and older. The ability to flex the elbow either actively or passively is of
great assistance in activities of daily living.
 Van Heest A, Waters PM, Simmons BP: Surgical treatment of arthrogrypsosis of the elbow. J Hand Surg Am
1998;23:1063-1070.
 Caroll RE, Hill NA: Triceps transfer to restore elbow flexion: A study of fifteen patients with paralytic lesions and
arthrogryposis. J Bone Joint Surg Am 1970;52:239-244.

100.A 14-year-old competitive gymnast has had activity-related low back pain for the
past month. Examination reveals no pain with forward flexion, but she has some
discomfort when resuming an upright position. She also has pain with extension
and lateral bending of the spine. The neurologic examination is normal. Popliteal
angles measure 20 degrees. AP, lateral, and oblique views of the lumbar spine are
negative. What is the next most appropriate step in management?

1- Referral to a pain clinic


American Academy of Orthopaedic Surgeons
2004 pediatric orthopedic Surgery

2- MRI
3- CBC, erythrocyte sedimentation rate, and C-reactive protein
4- Single photon emission computer tomography (SPECT)
5- Electromyography and nerve conduction velocity studies

PREFERRED RESPONSE: 4

DISCUSSION: Symptoms of activity-related low back pain, physical findings of pain with
extension, lateral bending, and resuming an upright position, and relative hamstring tightness are
consistent with spondylolysis. While the initial diagnostic work-up should include plain
radiographs of the lumbosacral spine, the findings may be negative because it can take weeks or
months for the characteristic changes to become apparent. SPECT has been a useful adjunct in
the diagnosis of spondylolysis when plain radiographs are negative. Since the patient's pain is
activity related and she is otherwise healthy, evaluation for infection is not indicated. Because
the neurologic examination is normal, electromyography, nerve conduction velocity studies, and
MRI are not indicated. CT can be used in those instances in which SPECT and bone scans are
negative.
 Ciullo JV, Jackson DW: Pars interarticularis stress reaction, spondylolysis, and spondylolisthesis in gymnasts. Clin Sports
Med 1985;4:95-110.
 Collier BD, Johnson RP, Carrera GF, et al: Painful spondylolysis or spondylolisthesis studied by radiography and single
photon emission computed tomography. Radiology 1985;154:207-211.
 Jackson DW, Wiltse LL, Cirincione RT: Spondylolysis in the female gymnast. Clin Orthop 1976;117:68-73.
 Ginsberg GM, Bassett GS: Back pain in children and adolescents: Evaluation and differential diagnosis. J Am Acad Orthop
Surg 1997;5:67-78.

Potrebbero piacerti anche